Medicine Ii
Medicine Ii
2
14.A 70-years old, known case of parkinsonism for 22 years presented d. Guillain Barre syndrome
in emergency with headache and confusion for last 2 weeks. He e. Diabetic neuropathy
had history of fall 3 days prior to this condition. GCS 10-11/15,
plantar are down going, with no neck rigidity. What is likely
diagnosis? (18LM Annual Exam)
a. Bacterial meningitis
b. Hemorrhagic stroke
c. Hydrocephalus
d. Subdural hematoma
e. Subarachnoid hemorrhage
20.A 36-years young diabetic for last 04 years is alcoholic having had
a respiratory tract infection two weeks ago, had develop a loss of
power in his both lower limbs since last 02 days. On examination
there is areflexia in legs with intact sensation. CSF examination
reveals increased proteins most possible diagnosis will be:
a. Alcoholic neuropathy
b. Acute transverse myelitis
c. Cord infarction
3
21.Recognized clinical feature of Parkinson's disease:
a. Intention tremors
b. Rigidity
c. Ataxia
d. Waddling gait
e. Decreased tone
4
29.A 33-years old male presented with headache, fever and e. Proximal median nerve lesion
drowsiness for 05 days. On examination there is bilateral optic disc
edema and extensor planter response. What is your diagnosis? 37.An old man on examination has Right inferior
a. Leuko-encephalopathy homonymous quadrantanopia. Where is the lesion?
b. Viral encephalitis a. Optic chiasm
c. Hepatic encephalopathy b. Right temporal lobe
d. Devic's disease c. Left optic tract
e. Subacute combined degeneration of cord d. Right occipital lobe
e. Left parietal lobe
30.Which of the following statement is CORRECT regarding lesion of
Vermis of the cerebellum? 38.A 20-years old man lacerated his medial aspect of his
a. Gait is normal wrist, damaging the ulnar nerve. What is he at risk of
b. Unable to walk heel to toe developing?
c. Dysmetria a. Inability to pinch paper between his thumb & index finger
d. Heel shin test is positive b. Claw hand
e. Nystagmus is positive c. Wasting of the 2nd lumbrical hand
d. Loss of sensation over the lateral three & a half fingers
31.Which of the following feature suggestive of Partial e. Wasting of the thenar eminence
complex seizures?
a. Aura 39.Which one of the following is most typical of frontal
b. Extensor planter response lobe dysfunction?
c. Atonia a. Visual field defects
d. Dilated pupil b. Inability to draw a clock face
e. Insomnia c. Inability to perform serial 7s
d. Sensory inattention
32.What is a likely cause of the meningitis if CSF DR shows e. Inability to generate a list rapidly
increased lymphocyte count and low sugar?
a. Meningitisim 40.Optic ataxia refers to difficulty in which of the following?
b. H. influenza meningitis a. Controlling hand eye coordination
c. Tuberculous meningitis b. Shifting gaze
d. Brain abscess c. Color vision
e. Streptococcal meningitis d. Moving the eyes T
e. Facial recognition
33.A 22-years old male presented with high grade fever and
neck stiffness since last 04 days. The ideal treatment would 41.Which of the following is a recognized cause of phrenic nerve palsy?
be cephalosporin to treat the condition is: a. Dermoid
a. Cephalexin b. Ganglioneuroma
b. IV amoxicillin c. Pericardial cyst
c. Gentamicin d. Sarcoidosis
d. IV ceftriaxone e. Aortic aneurysm
e. Cefpodoxime
42.Following are the clinical features of lower motor neuron
34.A 54-years old female is admitted with progressive weakness (LMN) lesion, EXCEPT:
three weeks following a trivial flu like illness. Which of the a. Atrophy of muscle supplied
following would make Guillain Barre syndrome an UNLIKELY b. Flaccid paralysis
diagnosis? c. Increased muscle tone
a. Autonomic dysfunction d. Muscular fasciculation
b. Sensory loss below T1 e. Loss of reflexes
c. Hyporeflexia
d. Elevated protein on CSF examination 43.A 63-years old smoker known case of diabetes, hypertension and
e. Marked muscle wasting atrial fibrillation underwent a dental filling procedure. Although the
procedure was uneventful, that night he experienced the acute
35.Which one of the following drugs would be responsible onset of vertigo and vomiting. The patient noted imbalance and
for Idiopathic intracranial hypertension? incoordination. Neurologic examination revealed a conscious, alert
a. Vitamin A supplement patient with gait ataxia and no other neurologic deficits. General
b. Erythromycin physical examination and systemic examination were normal. A
c. Topical tetracycline diagnosis of posterior circulation ischemic stroke was made. What
d. INH are the risk factors of stroke in this patient?
e. Ampicillin a. Smoking
b. Diabetes
36.A 25-years old male presents to the emergency department with c. Atrial fibrillation
weakness of his right hand. Examination reveals weakness of d. Hypertension
right wrist and finger extension. What is the likely diagnosis? e. All of the above
a. Axillary nerve palsy
b. Radial nerve lesion
c. Ulnar nerve lesion at the elbow
d. C8 nerve root lesion
5
44.An 18-years old male admitted in medical ward with history of 51.A 51-years old male presented with a main complaint of severe,
high- grade fever and unconsciousness for last two days. On sharp, piercing intermittent electrical like shock pain on the left side
examination his temperature was 99°F, GCS 7/15 and planters of his face. The pain was triggered by talking, washing his face,
bilaterally upgoing with neck rigidity positive. Which is the eating and brushing of his teeth. No pathology was detected during
investigation of the internal and external oral examination. What is your diagnosis?
choice in this case? a. Arthritis temporomandibular joint
a. CBC b. Molar pulpitis
b. Lumbar puncture & CSF analysis c. Temporomandibular joint dislocation
c. MRI brain d. Trigeminal neuralgia
d. Chest X-ray e. Temporal arteritis
e. CT scan brain
52.Regarding the risk factors of stroke and their prevention all are
45.A 30-years old male admitted in medical ward with history of high- true, EXCEPT:
grade fever, headache, vomiting and altered level of consciousness. a. Statins therapy (cholesterol lowering therapy)
On examination his temperature was 102°F, respiratory rate was b. Weight reduction in obesity
30 breaths/minute, GCS 10/15 and Kerning’s sign was positive. c. Heavy alcohol consumption
What is your diagnosis? d. Active life style
a. Pneumonia e. Treatment of hypertension is vital importance
b. Meningitis
c. Encephalitis 53.Transient ischemic attack is a type of stroke in which
d. Stroke neurological deficit reverses within:
e. Trauma a. 01 week
b. 24 hours
46.Which structure is most likely affected in Idiopathic c. 72 hours
Parkinson's disease? d. 01 month
a. Left cerebellar hemisphere e. 48 hours
b. Substantia nigra
c. Olfactory nerve 54.Which of the following is NOT related with Motor neuron disease?
d. Left cerebral neocortex a. It is progressive disorder of unknown cause
e. Red nucleus b. Usually present with upper & lower motor neuron signs
c. Usually occur after the age of 50 years
47.A 30-years old pregnant female, who spontaneously woke up with d. Sensory involvement is very common
the inability to voluntarily move the right side of her face, e. Speech & swallowing may be affected
including
the inability to close her right eye and slight drooping at the angle of 55.Regarding thrombolysis in acute ischemic stroke exclusion
her mouth. What is your diagnosis? criteria include:
a. Right facial nerve palsy a. Hemorrhage on pretreatment CT or MRI brain
b. Trigeminal nerve palsy b. Platelet count <100x109/liter
c. Left facial nerve palsy c. Blood pressure consistently > 185 systolic or more than
d. Oculomotor nerve palsy 110 diastolic
e. Stroke d. Active pancreatitis
e. All of the above
48.Which of the following is NOT a clinical feature of raised
intracranial pressure? 56.A 45-years old male known case of hypertension, diabetes
a. 6th cranial nerve lesion (false localizing sign) mellitus, coronary artery disease, dyslipidemia and smoking
b. Headache with vomiting presented in emergency department with right sided weakness
c. Bilateral extensor planter response and global aphasia. What is your diagnosis?
d. Papilledema a. Trauma
e. Tachycardia b. Stroke
c. Encephalitis
49.Signs of raised intracranial pressure include: d. Meningitis
a. Exaggerated reflexes e. None of above
b. Bradycardia
c. Miosis 57.A 50-years old man with history of ischemic heart disease present
d. Tremors with right side weakness with brief episode of loss of
e. Tachycardia consciousness. After regaining consciousness power was normal on
motor system examination and MRI brain was also normal. What is
50. Raised intracranial pressure can be reduced by: likely diagnosis?
a. Furosemide a. TIA (transient ischemic attack)
b. Mannitol b. Epilepsy
c. Normal saline c. Syncope
d. Ringer lactate d. Extrapyramidal disease
e. Haemaccel e. Hypoglycemia with transient disturbance of consciousness
6
58.Risk factors of stroke include all, EXCEPT: 67.A 67-years old man develops coma over a few minutes. He is
a. Low fibrinogen level unresponsive on arrival to the hospital. He has ataxic respirations
b. Diabetes mellitus and pinpoint reactive pupils. Oculocephalic reflexes are absent.
c. Polycythemia There is no motor response to noxious stimulation. What is the
d. Excess alcohol consumption most likely diagnosis?
e. Hyperlipidemia a. Pontine hemorrhage
b. Subarachnoid hemorrhage
59.Generalized seizure types include: c. Cerebellar hemorrhage
a. Absence seizures d. Basal ganglia hemorrhage
b. Myoclonic seizures e. Lobar intracerebellar hemorrhage
c. Tonic clonic seizures
d. Tonic seizures 68.A 74-years old woman develops occipital headache, vomiting
e. All of the above and dizziness. She has B.P 180/100 mmHg, pulse 70 b/min and
respiration 30/min. She is unable to sit or walk of unsteadiness.
60.Which of the following is NOT a modifiable risk factor of stroke? Over the next few hours, she develops a decline in her level of
a. Age consciousness. What is the most likely diagnosis?
b. Diabetes a. Cerebellar hemorrhage
c. Hypertension b. Subarachnoid hemorrhage
d. Hyperlipidemia c. Pontine hemorrhage
e. Obesity d. Basal ganglia hemorrhage
e. Hypertensive encephalopathy
61.Which of the following statement is TRUE regarding causes
of meningitis? 69.A 52-years old man with poorly controlled hypertension in the past,
a. Listeria monocytogenes can cause fungal meningitis presents with increasing headache, confusion and vomiting. His B.P
b. Neisseria meningitides & Streptococcus pneumonia can is 230/120 mmHg and there is papilledema. He has no focal
cause bacterial meningitis deficits. What is most likely diagnosis?
c. Mycobacterium tuberculous is not included in bacteria a. Hypertensive encephalopathy
d. Cryptococcus neoformans is not a cause of fungal meningitis b. Subarachnoid hemorrhage
e. Herpes simplex virus can not cause meningitis c. Arteriovenous malformation
d. Hydrocephalus
62.The contraindication of lumbar puncture includes: e. Intracerebellar hemorrhage
a. Meningitis
b. Encephalitis 70.A 27-years old woman complains of double vision and fatigue at the
c. Subarachnoid hemorrhage end of the day. Further history reveals difficulty in chewing food
d. Space occupying lesion of brain (Brain tumor) and some weakness in climbing stairs, symptoms improve with
e. Multiple sclerosis resting. O/E there is weakness in the eyelids, masticatory muscles
and thigh flexors. Her hand grip decreases with repetitive action.
63.Intracerebral hemorrhage can occur due to following There is no sensory abnormality and reflexes are normal. Which of
conditions, EXCEPT: the following is the most likely diagnosis?
a. Anticoagulant therapy a. Myasthenia gravis
b. Arteriovenous malformation b. Hypercalcemia
c. Metabolic myopathies c. Multiple sclerosis
d. Blood dyscrasia d. Thyroid storm
e. Thrombolytic therapy e. Meningeal lymphoma
64.Regarding the causes of polyneuropathy all are true, EXCEPT: 71.A 67-years old woman with hypertension presents with sudden
a. Guillain Barre syndrome onset of headache, vomiting and left sided weakness. On
b. Diphtheria examination she has weakness of the left face, hand and leg. B.P
c. Paraproteinemia of 180/100 mmHg. Which of the following is the most
d. Diabetes appropriate diagnostic test?
e. Becker's muscular dystrophy a. CT scan of brain
b. MRI
65.Which of the following is NOT a cause of polyneuropathy? c. CSF examination
a. Parkinsonism d. U/S
b. Heavy metals in toxication e. X-ray
c. GBS (Guillain Barre syndrome)
d. Deficiency of B1 & B12 72.A 40-years old woman complains of episodes of severe unilateral
e. Diabetes stabbing facial pain that is intermittent for several hours and then
disappears for several days. Physical examination is entirely
66.Fever with headache and photophobia is characteristic normal. Which of the following is the most likely diagnosis?
clinical features of: a. Trigeminal neuralgia
a. Normal pressure hydrocephalus b. Herpes zoster
b. Brain tumor c. Acoustic neuroma
c. Meningitis d. Bell's palsy
d. Benign intracranial hypertension e. Diabetic neuropathy
e. Subarachnoid hemorrhage
7
73.A 63-years old man developed a transient episode of vertigo, 80.Regarding the etiological pattern of paraplegia all are true, EXCEPT:
slurred speech, diplopia and paresthesias. He is symptom free now a. May be caused by TB spine
and b. May be caused by Parkinsonism
clinical examination is entirely normal. His past medical history is c. May be caused by spinal tumors
significant for HTN and dyslipidemia. Which of the following is d. May be caused by multiple sclerosis, combined degeneration
the most likely cause for symptoms? of cord
a. Posterior circulation TIA e. All of above
b. Anterior communicating artery aneurysm
c. Hypertensive encephalopathy 81.In Lateral medullary syndrome the clinical findings include:
d. Pseudobulbar palsy a. There is an ipsilateral facial numbness
e. Occlusion of the middle cerebral artery b. Diplopia (ipsilateral)
c. Spinothalamic sensory loss (contralateral)
74.Which of the following is the most likely finding in a 79-year- d. Horner's syndrome (ipsilateral)
old woman with Parkinson's disease? e. All of the above
a. Akinesia
b. Constant fine tremor 82.Ascending paralysis with areflexia is seen in patient of:
c. Muscle atrophy a. Diabetic motor neuropathy
d. Pupillary constriction b. Cervical cord compression
e. Spontaneous remission. c. Friedrich's ataxia
d. GB syndrome
75.A patient having sinusitis, face infection presented with high e. Subacute combined degeneration of cord
grade fever and fits. What is the most likely diagnosis?
a. Brain abscess 83.Regarding multiple sclerosis all are true, EXCEPT:
b. Encephalitis a. Relapsing & remitting MS is not a common clinical pattern
c. Meningitis b. Relapsing & remitting is common clinical pattern 80 to 90%
d. Brain tumor c. Primary progressive MS is 10 to 20% of cases
e. CVA d. Occasionally MS runs in a fulminating course less than 10%
of cases
76.Which of the statement is TRUE regarding meningitis? e. Secondary progressive usually follows on relapsing &
a. Viral meningitis is usually benign & self-limiting condition remitting disease
b. Petechial rash is never present in meningococcal meningitis
c. Meningitis cannot be caused by fungi 84.Regarding multiple sclerosis all are true, EXCEPT:
d. Acute pneumococcal meningitis should be treated a. MS is a chronic inflammatory disease of CNS
with rifampicin b. Precise mechanism is unknown
e. Intrathecal antibiotic should be used to treat acute c. Commonest age of onset in b/w 20 to 45 years
bacterial meningitis d. Relapsing & remitting MS is less common clinical type
e. Differential diagnosis includes SLE, sarcoidosis.
77.In epileptic woman with pregnancy which statement is CORRECT? Bechet’s syndrome
a. Carbamazepine can be given safely
b. Newer antiepileptic drugs like lamotrigine can be given 85.Multiple sclerosis is:
safely a. An infective disorder
c. Phenytoin is contraindicated b. Characterized by mono-neuropathy only
d. Sodium valproate is partially indicated c. A vascular disorder
e. Trimethadione is contraindicated d. Characterized by optic atrophy
e. Always progressive
78.Which one of the following is most likely CORRECT
statement regarding Normal pressure hydrocephalus? 86.A middle-aged man has presented with slowness of speech
a. CT/MRI brain reveal dilation of ventricles (communicating movements for last one month. He has noticed the change in his
hydrocephalus) voice tone too. On examination he is vitally stable and examination
b. It is due to progressive increase in CSF pressure of limbs shows rigidity. These features are due to the disease of:
c. Predominantly in person less than 50 years a. Cerebral cortex
d. Shunt procedure has no role in management b. Cerebellum
e. Urinary incontinence is late feature c. Extrapyramidal tract
d. Pyramidal tract
79.A 72-years old male presented with progressive memory loss, e. Posterior column
incontinence and walking difficulty. On examination mini
mental scale was in favor of dementia and gait is abnormal 87.Following are the lower motor neuron signs, EXCEPT:
(apraxia) but planters were normal. Which of the following is a. Individual muscles are involved usually
most likely diagnosis? b. Absent reflexes (deep tendon)
a. Alzheimer's disease c. Fasciculations
b. Frontal lobe dementia d. Hypotonia
c. Leavy body dementia e. Babinski's sign +ve
d. Multi infarct dementia
e. Normal pressure hydrocephalus
8
88.Following are the extrapyramidal disorders, EXCEPT: 97.The cranial nerve commonly involved in T.B meningitis is:
a. Chorea a. Optic nerve
b. Myasthenia gravis b. Defecting nerve
c. Parkinsonism c. Abducens nerve
d. Tremors d. Trigeminal nerve
e. Athetosis e. Vagus nerve
89.Intentional tremors and nystagmus are the features of: 98.Bradycardia, rigidity and tremors are typical feature of:
a. Cerebral lesion a. Encephalitis
b. Cerebellar lesion b. Motor neuron disease
c. Basal ganglia lesion c. Multiple sclerosis.
d. Medulla oblongata lesion d. Parkinsonism
e. Pons lesion e. Peripheral neuropathy
94.Facial nerve palsy is characterized by: 103. Which one of the following is the drug of choice in
a. Miosis absence seizures in childhood?
b. Ptosis a. Ethosuximide
c. Drying of angle of mouth b. Carbamazepine
d. Loss of wrinkling of forehead c. Phenobarbitone
e. Loss of taste over posterior 2/3 of tongue d. Phenytoin
e. Diazepam
95.Severe unilateral episode of headache with vomiting is usually
seen in patient with: 104. A 68-years old male complaining of sudden onset weakness of
a. Brain tumor right upper limb. On examination there is flaccid paralysis and
b. Encephalitis impaired sensation affecting right upper limb. Physical
c. Meningitis examination is normal. Which of the following will be the most
d. Migraine probable diagnosis?
e. Tension headache a. Partial anterior circulation syndrome
b. Focal seizures
96.Friedrich’s ataxia is associated with: c. Posterior circulation stroke
a. Diabetes mellitus d. Total anterior circulation stroke
b. Diabetes insipidus e. Subacute combined degeneration of spinal cord
c. Addison's disease
d. Cushing syndrome
e. Hypothyroidism
9
105. A 24-years old female diagnosed as a case of epilepsy 113. CT-scan brain has limited ability to image:
presented with seizures. Which one of the following seizures types a. Cysts
is most commonly seen in adults? b. Tumors
a. Complex partial c. Brain abscess
b. Generalized tonic clonic d. Brain hemorrhage
c. Generalized absence e. Posterior fossa structures like cerebellum brain stem
d. Simple partial
e. Jacksonian 114. An obese 55-year-old man presents with
mononeuritis multiplex. He is most likely to have:
106. A 50-years old male diagnosed as a case of myasthenia a. Infectious mononucleosis
gravis. Which one of the following is the most common b. Lead Poisoning
manifestation of muscle weakness? c. Guillain-Barre syndrome
a. Diaphragmatic weakness d. Diabetes mellitus
b. Wrist-drop e. Myxedema
c. Foot drop
d. Ocular muscle weakness 115. A known diabetic patient has developed vesicular rash with
e. Dysphagia burning sensation below the left nipple extending to the back along
with his cage since last three days. The likely diagnosis is:
107. A 24-years old female patient presents with unilateral a. Herpes simplex
headache, photophobia, vomiting and numbness. Which one of the b. Herpes zoster
following is the best treatment option? c. Drug reaction
a. Beta blockers (prophylaxis for frequent, > 2 episodes/month) d. Chicken pox
b. Sumatriptan (for severe attack abortion) e. Scarlet fever
c. NSAIDS & metoclopramide (for acute attack & nausea)
d. Steroids 116. A 55-years male presents with a diagnosis of right sided
e. Calcium channel blockers stroke. He is hypertensive. Before start of treatment which of
following investigation you should do early?
108. An 18-years old female student presents with fever and a. CT Brain
headache. On examination she is conscious but has neck b. EEG
stiffness. The CSF gram stain shows intracellular gram-negative c. PT & APTT
diplococci. The most probable diagnosis is: d. Lumber Puncture
a. Meningococcal meningitis e. Lipid profile
b. Hemophilus influenza meningitis
c. Streptococcus pneumonia meningitis 117. A 19-year-old man is admitted with a GCS of 12. He was doing
d. Listeria monocytogenes pushups when he complained of a sudden-onset, severe
e. E. coli meningitis headache and collapsed. What would you expect?
a. Subarachnoid hemorrhage
109. Regarding stroke (cerebrovascular accident) all are b. Fit
true, EXCEPT: c. Hypoglycemia
a. Most common stroke is caused by thromboembolic infarction d. Vasovagal syncope
b. Clinical features depend on infarct site & size e. TIA
c. It is the 10th common cause of death in developed countries
d. Stroke is uncommon below the age of 40 years 118. A 70-years old patient presented with pyramidal rigidity.
e. Hypertension is a modifiable risk factor Which of following is most likely sign?
a. Hypertonia
110. Following are the types of generalized seizures, EXCEPT: b. Bradykinesia
a. Absence seizures c. Ataxia
b. Myoclonic seizures d. 3rd nerve palsy
c. Tonic clonic seizures e. Sensory deficit
d. Tonic seizures
e. Jacksonian seizures 119. A 36-year-old woman presents to clinic with neurological
symptoms. On examination, she is able to stand with her feet
111. Which is NOT true regarding Myasthenia gravis? together. Upon closing her eyes, however, she is unable to keep
a. Characterized by drooping of upper eyelids her balance. What is the diagnosis?
b. Usually muscle weakness & fatigue occur on exertion a. Sensory ataxia
c. This is an autoimmune disorder b. Cerebellar tumor
d. It is presynaptic neuromuscular junction disorder c. Alcohol abuse
e. It is postsynaptic neuromuscular junction disorder d. Motor neuron disease
e. Parkinson disease
112. Regarding clinical features of transient ischemic attacks
of posterior circulation (TIA): which is NOT included?
a. Ataxia
b. Dysarthria
c. Diplopia, vertigo
d. Tetra paresis
e. Apraxia
1
0
120. A 60-years old patient presented with extrapyramidal 128. A 45 years male presented with urinary incontinence,
rigidity. Which of following is most likely sign? Anosmia and contralateral hemiparesis and leg is more affected
a. Bradykinesia than arm. Which artery is involved?
b. Hypotonia a. Middle cerebral artery
c. Ataxia b. Anterior cerebral artery
d. 3RD nerve palsy c. Basilar artery
e. Sensory deficit d. Posterior cerebral artery
e. Vertebral artery
121. A 30-years old male presented with rigidity and
upgoing plantars, which of following is most likely sign 129. A 17-year-old girl is brought into accident and emergency with
suggestive of multiple sclerosis? generalized tonic clonic seizure. Her mother found her fitting in her
a. Glabellar tap bedroom about 20 minutes ago. She has received two doses of
b. Bradykinesia rectal diazepam but did not improve. What is the next management
c. Ataxia step?
d. Sensory deficit a. Lorazepam
e. Combined 3rd and 6th nerve palsy b. Phenobarbital
c. Intubation
122. 22-year-pregnant lady comes for treatment of d. Call ICU
generalized seizures. What is the most appropriate drug? e. Phenytoin loading
a. Levetiracetam
b. Phenobarbital 130. A dissociated sensory loss may be seen in:
c. Ethosuximide a. Syringomyelia
d. Clonazepam b. Anterior spinal artery occlusion
e. Lamotrigine c. A radiculopathy
d. Occlusion of a middle cerebral artery
123. A 15-year-old male patient present in OPD with history of e. Compression of the spinal cord by a prolapsed
paraparesis of one week duration. Which of the following intervertebral disc
feature suggesting poliomyelitis?
a. Asymmetric flaccid paralysis 131. A 35-year male presented with spastic paraparesis, reduced
b. Symmetric flaccid paralysis vibration and proprioception sense in both lower limbs. His
c. Sensory involvement CBC reveals macrocytic anemia. Which of the following sign will
d. Involves distal muscles only you observe?
e. Bowel bladder dysfunction a. Optic neuritis
b. Romberg sign
124. Duration of TIA (Transient ischemic attack) in c. Glabellar tap reflect
a. Less than 1 week d. Pin point pupil
b. More than 24 hours e. Loss of proprioception
c. Less than 24 hours
d. More than a month 132. Inability to perform alternating movement of the hands
e. Less than 72 hours is referred to as:
a. Dysdiadochokinesia
125. The Lateral medullary syndrome is caused by occlusion of b. Dyskinesia
which one of the following blood vessels? c. Romberg's sign
a. Anterior inferior cerebellar artery d. Chorea
b. Posterior cerebral artery e. Athetosis
c. Lateral sinus thrombosis
d. Middle cerebral artery 133. A young male presented with weakness of legs after
e. Posterior inferior cerebellar artery taking canned food, what will be diagnosis?
a. Botulism
126. A surgical patient is diagnosed with tetanus. The b. Tetanus
following feature differentiate from Generalized tonic-clonic c. Myelitis
seizures: d. Polio
a. Frothing from mouth e. Multiple sclerosis
b. Opisthotonus posture
c. Laryngeal spasm 134. Choreatic movements are:
d. Aura a. Slow and writhing.
e. Urinary incontinence b. Shock-like asymmetrical and irregular.
c. Brief, jerky and irregular
127. A 60-years old Patient presented with walking problem, d. A sign of restlessness
urinary incontinence and loss of memory. After CT brain shows e. Rhythmical and oscillatory
dilated ventricles. Which of following investigation would you
advise?
a. MRI Brain
b. EEG
c. PT & APTT
d. Lumber Puncture
e. Lipid profile
1
1
135. A 30 years male Patient presented with head ache and fits 142. For the patient with the history of fever, headache and
since 2 years. CT scan shows a mass in frontal lobe, which of neck stiffness the most important investigation is:
following sign is suggestive for raised intracranial pressure a. CSF examination
a. Hypotonia b. CBC
b. Papilledema c. CT-scan brain
c. Ataxia d. MRI brain
d. Hypotension e. X-ray skull
e. Sensory deficit
143. The gait of the patient with cog-wheel rigidity and pill-
136. A 24-years old boy brought in emergency department with rolling tremor is likely to be:
history of headache, fever, altered mentation for one month with a. Drunken
right sided ophthalmoplegia, positive neck rigidity. CSF-DR b. Hemiplegic
shows cells: 200, mostly lymphocytes, normal glucose, high c. High stepping
proteins, pressure slightly elevated. What is the likely diagnosis? d. Normal
a. Tuberculous meningitis e. Shuffling
b. Cerebral meningitis
c. Aseptic meningitis 144. The gait of patient with nystagmus and resting tremors is
d. Fungal meningitis likely to be:
e. Viral meningitis a. Drunken
b. Hemiplegic
137. Recognized causes of motor neuropathy include: c. High stepping
a. Diabetes mellitus d. Ataxic
b. Porphyria e. Shuffling
c. Guillain-Barre syndrome
d. Friedrich’s ataxia 145. A 20-year-old male had brief episode of altered level of
e. Diphtheria consciousness associated with hallucinations and a rising epigastric
sensation, diagnosed as a case of complex-partial seizure with
138. A sudden development of bilateral external ophthalmoplegia temporal origin. Which of the drugs is best for this type of seizures
is NOT associated with? disorder?
a. Botulism a. Carbamazepine
b. Myasthenia gravis b. Paroxetine
c. Wernicke’s encephalopathy c. Valproic acid
d. Neurosyphilis d. Haloperidol
e. Stroke e. Phenobarbitone
139. A 37-years old man presents with visual impairment. 146. The cortical blindness and visual agnosia are caused
Examination reveals; a bitemporal hemianopia. Which of by occlusion of which one of the following blood vessels?
the following tumors is most likely responsible for this a. Anterior inferior cerebellar artery
finding? b. Posterior cerebral artery
a. Optic glioma c. Lateral sinus thrombosis
b. Occipital astrocytoma d. Middle cerebral artery
c. Brainstem glioma e. Posterior inferior cerebellar artery
d. Pituitary adenoma
e. Sphenoid wing meningioma 147. Which of the following is an example of a peripheral
sensory polyneuropathy?
140. An old man presented in emergency ward with history of a. Intermittent claudication
weakness of right side of body of rapid onset. The most helpful b. ‘Stocking glove’ loss of sensation
first line investigation for management of this patient is: c. Oculomotor palsy
a. Cerebral angiography d. Priapism
b. CSF examination e. Thick radial nerve
c. CT-scan brain
d. Fasting lipid profile 148. Regarding classic migraine headache:
e. Nerve conduction study a. Unilateral headache with fever
b. Without aura
141. A 55-year-old man presented with progressive weakness of his c. Associated with aura
hands over a period of one year. Clinical examination reveals d. Common cause of paralysis
wasting of the muscles of the hands and forearms and e. It increases with cough, stooping position
fasciculations with hyperreflexia of his lower limbs and upgoing
planters. Sensory examination was normal. The most likely 149. Regarding TIA (transient ischemic attack):
diagnosis is: a. Neurological defect occurs sudden and remains less than 24
a. Alzheimer’s disease hours
b. Motor neuron disease b. Defect remains less than 7 days
c. Multiple cerebral infarcts c. It is treated with antibiotic
d. Multiple sclerosis d. It is normal in aged person
e. Syringomyelia e. Patient has no future chance of stroke
10
150. Guillain-Barre syndrome:
a. It is ascending type paralysis
b. It is upper motor neuron type lesion
c. It is treated by high dose of I/V fluids
d. I/V corticosteroid is 1st choice
11
INFECTIOUS DISEASES 8. You are questioned by your registrar regarding bacteria
1. A 15-year-old school boy presented with fever, headache and responsible for causing blood-stained diarrhea. From the list below
bilateral tender and enlarged parotid glands. After 10 days his right select the
tested was swollen. The most likely diagnosis is: (18LM Annual Exam) organism which is NOT responsible for causing blood-stained
a. Gonococcal infection diarrhea:
b. Mumps a. Campylobacter spp.
c. Sarcoidosis b. Shigella spp.
d. Sjogren syndrome c. Escherichia coli
e. HIV d. Staphylococcus spp.
e. Salmonella spp.
2. A 32-year-old male has presented with high-grade fever and
severe body ache since last 03 days. On examination pulse is 9. Which one of the following antimalarial drugs will eliminate
106/m, temp 104F, there is no spleen or lymph nodes palpable. tissue schizonts of the malarial parasite?
His leg examination shows reddish spots. Blood CP shows: a. Primaquine
Hb—8.8 gm% WBC b. Chloroquine
c. Proguanil
—3200 c/mm3
d. Quinine
Platelets— 61000 c/mm3 e. Pyrimethamine
The likely diagnosis is: (18LM Annual Exam)
a. Brucellosis 10. A bus driver developed high grade fever, headache and severe body
b. Dengue fever ache. O/E his eyes were red and maculopapular rash all over body
c. Malaria except palms and soles. Platelets were 80000 c/mm3. Diagnosis
d. Typhoid will be:
e. DIC a. Malaria
b. Enteric fever
3. The drug used in highly prevalent drug-resistant falciparum c. Meningococcemia
malaria is? (18LM Annual Exam) d. Sepsis
a. Chloroquine e. Viral hemorrhagic fever
b. Primaquine
c. Mefloquine 11. A HIV positive pregnant lady came to your clinic. She is very much
d. Doxycycline tensed about risk of transmission of HIV to her child. What will
e. Proguanil you advise?
a. Therapeutic abortion because of high risk of transmission
4. The drug used for chemoprophylaxis in highly prevalent b. Continue pregnancy, no risk of transmission
drug- resistant falciparum malaria is: c. Avoid breast feeding
a. Chloroquine d. Zidovudine during pregnancy and after delivery
b. Primaquine e. Lamivudine in last trimester
c. Mefloquine
d. Doxycycline 12. The best diagnosis of dengue fever within five days of
e. Proguanil infection would be?
a. NS1 antigen
5. The following gastrointestinal infection can lead to b. PCR
reactive arthritis: c. Dengue IgM
a. Pseudomonas d. CBC
b. Shigella, salmonella e. Dengue IgG
c. Chlamydia trachomatis
d. Ureaplasma urealyticum 13. Which is FALSE regarding Pneumocystis carinii pneumonia in AIDS?
e. E. Coli a. Once a patient has had it, they are unlikely to get it again
b. Prophylaxis should be given in all patients with CD4 count <200
6. Which organism is most commonly responsible c. CXR may be normal in up to 20% of patients
for pseudomembranous colitis? d. It is usually only seen when the CD4 count
a. Rotavirus e. Characteristically shows bilateral diffuse infiltrates <200
b. Salmonella
c. Clostridium difficile 14. Which is NOT an AIDS defining illness:
d. Toxicogenic strain of E. coli a. Esophageal candidiasis
e. Vibrio cholera b. Pulmonary TB
c. Kaposi sarcoma
7. The bacterial causes of acute infectious diarrhea are: d. Toxoplasmosis
a. Entamoeba histolytica e. Herpes zoster
b. Giardia lamblia
c. Rota virus 15. The 40-years old male presented with painless ulcer at glans
d. E. coli penis after unprotected sex. This lesion is:
e. H. pylori a. Genital wart
b. Pemphigus
c. Balanitis
d. Granuloma inguinale
e. Chancre
12
16. Which does not require post exposure prophylaxis for rabies? 24. A 28-years old pregnant female presented with third trimester
a. Mucous membrane exposure to a bat with history of high-grade fever since 02 days. Thin film smear is
b. Scratch positive for Plasmodium falciparum. The appropriate treatment for
c. Bite on extremity this
d. Bite on face patient is:
e. Skin contact with blood, urine & faeces a. Chloroquine
b. Quinine
17. Which of the following disease needs skin test for diagnosis: c. Halofantrine
a. Diphtheria d. Primaquine
b. Brucellosis e. Artemether
c. Syphilis
d. Tuberculosis 25. Treatment of choice for schistosomiasis is:
e. Polio a. Prednisolone
b. Praziquantel
18. Which agent should NOT be a part of the management c. Doxycycline
of generalized tetanus? d. Albendazole
a. Labetalol e. Trimethoprim
b. Metronidazole
c. Penicillin 26. What is the first line treatment in Hydatid disease?
d. Tetanus immunoglobulin a. Itraconazole
e. Midazolam b. Ciprofloxacin
c. Albendazole
19. A 30-years old male presented with high grade fever, rigors d. Sodium stibogluconate
and mild splenomegaly. The diagnosis will depend on: e. Metronidazole
a. Typhi dot
b. Stool culture 27. A sexually active female presents to sexual disease clinic
c. Thick & thin smear for malaria with multiple painful genital ulcers. What is the likely cause?
d. Mycoplasma serology a. Treponema pallidum
e. Bone marrow biopsy b. Chlamydia trachomatis
c. Herpes simplex
20. A person was infected by typhoid fever 01 year back and still d. Primary HIV infection
his stool contains the salmonella typhi. This person is labelled e. Haemophilus ducreyi
as:
a. Active infection 28. A 17-years old girl is diagnosed with infectious
b. Chronic career mononucleosis. Which of the following is most appropriate
c. Resistant to antibiotic test to provide a diagnosis?
d. A case of non-responder a. Blood film
e. A case of relapse b. Blood culture
c. Lymph node biopsy
21. Which of the following infection is NOT transmitted by d. Mast cell tryptase
blood transfusion? e. Heterophil antibody test
a. Malaria
b. HCV 29. Which one of the following tests is most likely to remain positive
c. Schistosomiasis in a patient with syphilis despite treatment?
d. HIV a. Treponema pallidum hem-agglutination test (TPHA)
e. Syphilis b. Blood culture
c. Venereal disease research laboratory (VDRL)
22. A 29-years old woman develops severe vomiting 04 hours d. Rapid plasma reagin (RPR)
after having lunch at a local restaurant. What is most likely e. Wassermann reaction
causative organism?
a. Salmonella 30. Which of the following virus is thought to be the cause of
b. Escherichia coli Kaposi sarcoma?
c. Shigella a. HHV-8
d. Campylobacter b. HIV-2
e. Staphylococcus aureus c. CMV
d. HPV-8
23. What is the highest likelihood that a doctor acquires HIV from e. HTLV-1
a needle stick injury from an infected patient?
a. 30% 31. HIV patient is likely to develop following CNS disease:
b. 10% a. Creutz-Field Jacob's disease
c. 0.30% b. Piron disease
d. 05% c. Cerebral toxoplasmosis
e. 02% d. Cerebral malaria
e. Parkinson's disease
13
32. A middle-aged man used to walk on walking tract in evening before 39. A 24-year-old male known case of HIV presents with chronic
sunset. After 06 days of walking, he developed high grade fever diarrhea. Colonic biopsy reveals "owl's eye" appearance. The
with headache, backache and arthralgia. CBC reveals leucopenia likely causative organism is:
and a. Cytomegalovirus
thrombocytopenia. What next investigation will you advise and b. Cryptosporidium
establish the diagnosis? c. Clostridium difficile
a. ICT & immune chromatography for malaria d. Salmonella
b. Thick, thin blood film for malaria e. Shigella
c. Dengue antibodies IgG
d. Blood culture 40. A 20-years old male presented with fever, retrobulbar pain,
e. Typhi dot extreme myalgias and bleeding from nose and gums for 02 days.
Laboratory report shows WBC 1000x10/cmm and platelets
33. For prevention of malaria, you will advise to adopt 3000x10/cmm.
following measures, EXCEPT: What is the diagnosis?
a. Use mosquito replants a. Malaria
b. Use mosquito nets b. Dengue fever
c. Use nets on the windows & doors c. Hepatitis
d. Use primaquine for chemoprophylaxis d. Enteric fever
e. Use chloroquine for chemoprophylaxis e. Meningococcemia
34. A 22-years old male patient living in Dadu, presented with fever 41. Which of the following is common feature of infectious mononucleosis?
and rigors and unconsciousness for last two days. He is passing a. Arthritis
blackish urine since last night. The most likely suspected organism b. Agranulocytosis
is: c. Glomerulonephritis
a. Mixed Plasmodium falciparum & oval d. Pericarditis
b. Mixed Plasmodium falciparum & vivax e. Severe pharyngeal edema
c. Plasmodium falciparum
d. Plasmodium knowlesi 42. A 25 years old military recruit presented with fever, dry cough and
e. Plasmodium vivax maculopapular rash all over the body. On examination he is pale
and jaundiced. He was diagnosed as a case of Mycoplasma
35. A young patient presented with painless red plaque and pneumonia. What is the cause of jaundice in this patient?
superficial ulceration of glans penis with backache after sex. What a. Acute viral hepatitis
is your diagnosis? b. Cold agglutinin hemolytic anemia
a. AIDS c. Drug induced live disease
b. Syphilis d. Sickle cell trait
c. Psoriasis e. G6PD deficiency
d. Behcet's disease
e. Reiter's syndrome 43. A 19-years old college student develops a severe sore throat,
cervical lymphadenopathy and atypical lymphocytes on blood film.
36. A 20-years old male travelled from Karachi two days back A heterophil antibody test is positive. What is most likely diagnosis:
presented with generalized body ache, intermittent fever and a. Infectious mononucleosis
throat pain. His CBC shows lymphopenia while MP, blood culture, b. Hyperthyroidism
urine DR and c. ALL
chest X-ray was normal. The most likely diagnosis is: d. AML
a. Coronavirus infection e. Renal cell carcinoma
b. Acute rhinitis
c. Sinusitis 44. A 36-years old female presents with diarrhea and lower
d. Influenza abdominal pain, which has been ongoing for several weeks. Her
e. Pharyngitis stools were initially loose however the diarrhea is now bloody and
there is
37. A 20-year-old lady presented with high grade fever since 05 days. evidence of mucous. A stool sample contains trophozoites and
She also complaints of headache, myalgia, diarrhea and vomiting. On leukocytes. What is the likely organism leading to this condition:
examination temperature 103F, pulse 80/min, respiratory rate 18 a. Entamoeba histolytica
breath/min. The most suitable investigation for diagnosis is: b. Clostridium difficile
a. Widal test c. Giardia lamblia
b. Typhi dot d. Adenovirus
c. Urine culture e. Salmonella
d. Blood culture
e. Stool culture 45. A 20-years old woman develops a fever of 39 °C and severe myalgia.
Examination shows petechiae on the lower legs. A blood film is
38. A 24-year-old male presented with diarrhea since 02 days. negative for malarial parasites. The WBC count is at the lower limit
The likely cause of diarrhea is: of normal and the platelet count is 35x109/L. What is the most likely
a. Salmonella diagnosis?
b. E. coli a. Plasmodium vivax malaria
c. Clostridium difficile. b. Plasmodium falciparum malaria
d. Giardia c. Salmonella typhi
e. Tuberculosis
14
d. Rickettsia typhi
e. Dengue fever
15
46. A 30-years old woman presents with fever, rash and cough and is e. Varicella zoster virus
pyrexial, tachycardiac and tachypneic. She has a florid
erythematous rash on her face, trunk and arms with scattered
whitish papular lesions on the buccal mucosa. What is the most
likely diagnosis?
a. Meningococcemia
b. Rubella
c. Parvovirus B19
d. Secondary syphilis
e. Measles
48. A 10-years old child presented with sore throat with fever. On
examination marked tonsillar and pharyngeal exudation with
a yellow membrane. There was also cervical lymph nodes
enlargement. What is the diagnosis?
a. Mumps
b. Infectious mononucleosis
c. Diphtheria
d. Ludwig's angina
e. Pertussis
55. The most common cause of bleeding in viral hemorrhagic fever is:
a. Increased clotting time
b. Increased prothrombin time
c. Increased bleeding time
d. Increased capillary leakage & platelet dysfunction
e. Decreased von-Willebrand factor
17
62. The best indicator of immune decline in HIV patient is
a. HIV RNA
b. CD4 Count
c. P24 antigen
d. CD8 count
e. HIV antibody titer
18
ENDOCRINE AND METABOLIC DISEASES 7. A school teacher presented with tender swelling in front of
neck, moves to swallowing. TSH was suppressed with raised
1. The causes of secondary adrenocorticoid insufficiency include ESR. RAI uptake was low. Most likely she is suffering from:
all, EXCEPT: (18LM Annual Exam) a. Grave's disease
a. Sarcoidosis b. Thyroid adenoma
b. Surgical removal c. Hashimoto's thyroiditis
c. Amyloidosis d. De-Quervain thyroiditis
d. Autoimmune e. Thyroid carcinoma
e. Tuberculosis
8. A 23-years old woman presented with a painless swelling in her
2. An 18-year girl referred from gynecology OPD for physician neck. There was no history of a recent viral illness. The patient did
opinion with complaint of primary amenorrhea and short stature. not have symptoms of hyper or hypothyroidism. She was not taking
No any medicine. On examination 2cm painless mass (thyroid lump)
history of diabetes mellitus, celiac disease and thyroid disease. On associated with cervical lymphadenopathy. What is the most useful
examination short stature with webbing of neck and poorly investigation in elucidating the diagnosis?
developed secondary sexual characters. What is diagnosis? (18LM a. Thyroid ultrasound
Annual Exam) b. Radioiodine thyroid scan
a. Celiac disease c. Fine needle aspiration of the thyroid gland
b. Cystic fibrosis d. Total plasma thyroxine concentration
c. Down's syndrome e. CT scan chest
d. Diabetes mellitus
e. Turner’s syndrome 9. A patient with toxic multinodular goiter developed
hyperthermia and sore throat while undergoing medical
3. A 45-year-old woman brought in emergency unconscious. therapy. Which
On examination her pulse is 60 beats/minute, temperature medication could have caused these side effects:
94F, a. Prednisolone
thyroid gland was enlarged and ankle reflexes were delayed. How b. Carbimazole
will you further investigate? (18LM Annual Exam) c. Inderal
a. Obtain blood urea, creatinine d. Propranolol
b. Advise thyroid profile e. Potassium & magnesium aspartate
c. Advise for thyroid scan
d. Obtain blood culture 10. What statement is NOT correct relatively to the polycystic
e. Advise for CT scan brain ovarian syndrome?
a. Alopecia is common
4. Which of the following is the treatment of choice for patient b. Elevated androgen levels
with acromegaly? (18LM Annual Exam) c. Obesity is often found
a. Trans-nasal trans-sphenoidal pituitary surgery d. Patients with polycystic ovary syndrome have amenorrhea
b. Radiotherapy e. Hirsutism is common
c. Radiosurgery
d. Growth hormone receptor antagonist 11. A 45-years old male presented with abdominal pain, weight loss
e. Dopamine agonist and dark complexion progressively over 02 years. On examination
BP 90/50 mmHg. His blood sugar was 90 mg after meal. The most
5. A 42-year-old female presented with restlessness and palpitation likely diagnosis is:
for last 05 days. On examination; pulse is 106/min, BP 140/60 a. Paraneoplastic syndrome
mm Hg. Temperature 99F. Her tremor in the hands is positive. b. Melanin producing tumor
She had c. Colorectal carcinoma
enlarged and tender goiter. Last week she had a viral chest d. TB abdomen
infection. The likely diagnosis? (18LM Annual Exam) e. Addison's disease
a. Hashimoto thyroiditis
b. Subclinical hypothyroidism 12. A 55-years old male needs a diagnosis of pheochromocytoma
c. Subacute thyroiditis you will advise:
d. Multinodular goiter a. Pituitary sampling
e. Thyroid carcinoma b. CT scan of abdomen
c. MRI of adrenal
6. A 30-years old lady with past history of postpartum hemorrhage d. Urinary catecholamine levels
developed loss of axillary, pubic and body hairs. Later on, e. MIBG scan
noticed change of voice. Fasting glucose, serum FSH, LH were
low with 13. Which of the statement is false for acromegaly?
normal TSH. What is your diagnosis? a. The pituitary fossa is large on MRI
a. Hypothyroidism b. Associate with frequent headaches
b. Sheehan's syndrome c. Recession of lower jaw
c. Addison's disease d. There is abnormal glucose tolerance test
d. Hemochromatosis e. There is increase in heel pad thickness
e. Hypogonadism
19
14. Which is NOT a feature of hypothyroidism? d. Gonadotropin releasing hormone
a. Ophthalmopathy e. Prolactin
b. Weight gain
c. Bradycardia 23. In a patient with suspected insulinoma, which one of the
d. Constipation following is considered the best investigation?
e. Loss of outer third of the eyebrow a. Glucagon stimulation test
b. Supervised prolonged fasting
15. Which of the following drug can cause hyperthyroidism? c. Oral glucose tolerance test
a. Digoxin d. Early morning C-peptide levels
b. Flecainide e. Insulin tolerance test
c. Propranolol
d. Amiodarone 24. Which one of the following is the most common non-
e. Carbimazole iatrogenic cause of Cushing syndrome?
a. Pituitary tumor
16. Which is NOT a feature of Grave's disease? b. Ectopic ACTH production
a. Delayed tendon reflexes c. Adrenal adenoma
b. Atrial fibrillation d. Micronodular adrenal dysplasia
c. Hyperreflexia e. Adrenal carcinoma
d. Ophthalmoplegia
e. Systolic murmur 25. Which one of the following feature would most suggest a
diagnosis of Grave's disease?
17. Which of the following is screening test for Cushing syndrome? a. Multi-nodular goiter
a. IGFI (Insulin-like growth factor) b. Lid lag
b. Dexamethasone suppression test c. Pretibial myxedema
c. Urinary cortisol level d. Atrial fibrillation
d. Serum insulin level e. Family history of radioiodine treatment
e. ACTH
26. A 35-years old female is recently diagnosed with Addison's
18. A 22-years old female presented with amenorrhea and disease. Which of the following physical signs may you find in this
small breasts. What will you advise? patient?
a. Pelvic ultrasound a. Multiple bruises on her limbs
b. FSH & LH b. Caf
c. Serum estradiol c. Stretch marks on her abdomen
d. TSH d. Frontal balding
e. Serum cortisol e. Thinning of the axillary hair
19. A 30 years old patient presented with headache, infertility and 27. A 36-years old female with complaints of inability to lose weight
galactorrhea. On examination there was bitemporal despite of low-calorie diet and daily exercise. She has also
hemianopia. What is the most likely diagnosis? noticed cold intolerance, also reports constipation and hair loss.
a. Primary hypogonadism These symptoms have been worsening over past 02 to 03
b. Acromegaly months. An
c. Prolactin secreting tumor elevated TSH and low FT3 and FT4 confirms your diagnosis. What is
d. Neuro-sarcoidosis the best investigation to confirm the diagnosis?
e. Cushing syndrome a. Antinuclear antibodies
b. Thyroid peroxidase antibodies
20. Which one of the following is least characteristic of c. Radioactive iodide uptake
Addison's disease? d. Thyroid aspiration
a. Positive short ACTH test e. Thyroid ultrasound
b. Hyponatremia
c. Metabolic alkalosis 28. A 32-years old male has presented with progressive headache
d. Hyperkalemia and problems in vision since last 03 months. On examination he is
e. Hypoglycemia tall and having muscular body. His pulse is 78/minute, BP 150/90
mmHg. Laboratory investigations show blood sugar 230 mg. The
21. Which of the following is least recognized as a most appropriate investigation in this patient will be:
potential complication of acromegaly? a. ECG
a. Hypertension b. Fundoscopy
b. Cardiomyopathy c. MRI brain
c. Diabetes mellitus d. CT scan brain
d. Pulmonary hypertension e. Hemoglobin A1C
e. Colorectal cancer
29. In the treatment of hyperthyroidism:
22. Which one of the following hormones is under a. The maintenance dose of antithyroid drugs determined by
continuous inhibition? TSH only
a. Thyroid releasing hormone b. 10% remain thyrotoxic even after surgery
b. Growth hormones c. Radioiodine should be avoided in patients with active
c. Adrenocorticotropic hormone Grave's ophthalmopathy
11
0
d. Subtotal thyroidectomy is indicated for young patients
e. Antithyroid drugs are contraindicated during pregnancy
11
1
30. Bradycardia is a feature of: c. Carbimazole
a. Cushing syndrome
b. Hypothyroidism
c. Diabetes mellitus
d. Addison's disease
e. Hypoparathyroidism
32. A 40-years old female has presented with increasing weight and
lethargy for last 03 to 04 months. On examination patient is obese
with pulse 68/min, BP 160/100 mmHg and non-pitting edema. Her
cholesterol is 260 mg%. The likely cause of this hypertension is:
a. Diabetes mellitus
b. Acromegaly
c. Cushing syndrome
d. Hyperthyroidism
e. Hypothyroidism
11
2
d. Radioactive iodine
e. Surgery
11
3
45. A 58-years old woman has noticed dry skin, fatigue and weight gain c. Crohn's disease
over the past 03 months. Her B.P is 110/70 mmHg, pulse 60/min. d. Ketoconazole therapy
Serum electrolytes show Na; 130meq/L, cholesterol is raised, TSH e. Polyglandular syndrome
is elevated. What is the most common cause of her elevated TSH?
a. Primary hypothyroidism 52. Regarding the etiology of Cushing’s syndrome, the commonest
b. Trauma cause is:
c. Parathyroid surgery a. Adrenal adenoma
d. Antithyroid chemicals b. Ectopic ACTH syndrome
e. Radioactive iodine ingestion c. Cushing’s disease
d. Major depressive illness
What are the following causes of secondary osteoporosis, EXCEPT: e. Alcohol abuse
a. Hypogonadism
b. Cushing syndrome 53. A 24-year married lady was diagnosed case of thyrotoxicosis. She
c. Steroids was put on carbimazole. Now she has become pregnant. What is
d. Increased BMI (more than 30) the best option of her treatment?
e. Decreased BMI (less than 18) a. Continue carbimazole
b. Switch over methimazole
46. A 34-years old male diagnosed as a case of Cushing's syndrome c. Start propylthiouracil
admitted at ER for review. What are the following investigations are d. Advise iodine therapy
true, except? e. Advise propranolol
a. Failure to suppress serum cortisol with low doses of
oral dexamethasone 54. A 32 years female presented with infrequent menstruation,
b. Loss of normal circadian rhythm of cortisol with over growing of body hairs and abnormalities in ovaries which
inappropriately increased late-night serum or salivary cortisol of following tests will you advise?
c. Increased 24-hours urine free cortisol a. Serum cortisol
d. Increased serum cortisol b. Serum insulin and LH:FSH ratio
e. Decreased serum cortisol c. Serum estradiol
d. Serum testosterone
47. A 48-years old male diagnosed as a case of acromegaly. Which e. Serum growth hormone
one of the following is the first line treatment in acromegaly:
a. Trans-sphenoidal surgery 55. 30-year female presented with swelling in front of neck. Her lab
b. Radiotherapy reports are following, T4 raised and TSH found undetectable.
c. Somatostatin analogues Which of following is most likely diagnosis?
d. Dopamine agonists a. Primary thyrotoxicosis
e. GH receptor agonist b. Subclinical thyrotoxicosis
c. Non-thyroid illness
48. A 40-years old female presents to the outpatient department d. Subclinical hypothyroidism
with the complaint of weight gain, husky voice and poor hearing. e. T3, thyrotoxicosis
On examination there is non-pitting edema, carotenemia and
malar
56. 45-year female presented with H/O of increasing weight sine
flush. Which one of the following is best investigation for making the
last one year. She had history using anti-depressant drug. On
diagnosis:
CNS examination ankle jerk is delayed. What is your diagnosis?
a. Echocardiography a. Acromegaly
b. T3 & T4
b. Cushing syndrome
c. Serum cortisol c. Hypothyroidism
d. Anti-Ds DNA d. Hypoparathyroidism
e. TSH e. Obesity
49. Which of the following is the most common presentation of 57. Regarding the etiology; the commonest disorder leading to
anterior pituitary hyposecretion in a 26 years old woman? adrenal insufficiency is:
a. Amenorrhea
a. Sudden withdrawal of glucocorticoid therapy and pituitary
b. Decreased melanin pigmentation
tumour
c. Emaciation & cachexia
b. Tuberculosis
d. Loss of axillary & pubic hairs c. Bilateral adrenalectomy
e. Occurrence of myxedema
d. Drugs
e. Metastatic carcinoma
50. The features of myxedema coma include:
a. Hyperthermia
58. A 50-year-old presented with palpitation, excessive
b. Increase tendon reflexes
sweating, headache and tremor. On examination pulse
c. Ptosis 110/minute, B.P
d. Pericardial effusion
160/100 mmHg. Which one of the following investigations is best
e. Convulsions are common
for diagnosis?
a. Fasting blood sugar
51. Which of following disease is associated to low plasma cortisol b. ECG
and high ACTH level occurs? c. Echocardiography
a. Addison’s disease d. Serum electrolyte
b. Cushing’s disease
20
e. Urinary Metanephrine
20
59. Which of the following investigations is most useful for diagnosis 66. A young boy has presented with constipation and weight gain for
of acromegaly? last 2-months. On examination, pulse is 60/min, BP 140/90 mmHg.
a. Serum growth hormone His lips are swollen and non-pitting edema over feet is positive.
b. Estimation growth hormone during oral glucose tolerance test The likely abnormality in this patient is:
c. IGF-1 a. Decreased blood sugar
d. Serial measurements of blood sugar b. Decreased calcium
e. MRI c. Increased TSH
d. Decreased TSH
60. A 45-years female presented with centripetal obesity, acne e. Increased PTH
and hirsutism. Your diagnosis would be?
a. Cushing’s syndrome 69. A 20-years old female presented with short stature and
b. Diabetes mellitus oligomenorrhea. The diagnosis is made with Turner
c. Hypogonadism syndrome. Where will be defect in this disorder?
d. Hypothyroidism a. Deletion of BCR ABL translocation
e. Simple obesity b. Chromosome 10
c. Absence of X chromosome
61. Addison’s disease can be associated with: d. Chromosome 17
a. Hashimoto’s thyroiditis e. Mitochondrial abnormalities
b. Hyperparathyroidism
c. More intensive pigmentation 70. A 19-year-old man with history of tall stature, gynecomastia,
d. Vitiligo testicular atrophy is diagnosed as a case of Klinefelter’s syndrome.
e. Enlargement of the adrenal glands The chromosomal abnormality in this patient is:
a. XO
62. A 30-years old female presented in OPD with complaints of b. XY
swelling of neck associated with neck pain and dysphagia. On c. YY
examination swelling is tender and moves with process of d. XX
deglutition. Blood CBC shows leukocytosis. The most appropriate e. XXY
treatment would be:
a. NSAIDs 71. A 27-years old man is a suspected case of Klinefelter
b. Thioienes drugs syndrome. Which investigation is likely to be diagnostic:
c. Radioactive iodine a. Karyotype
d. Propranolol b. Prolactin level
e. Inorganic acid c. FISH analysis of DNA
d. PCR analysis of DNA
63. A 22-year-old female with Grave’s disease who requires e. MRI pituitary
antithyroid therapy becomes pregnant. What which anti-thyroid
drug is safest? 72. Regarding Klinefelter's syndrome all are correct, EXCEPT:
a. Potassium iodide (KI) a. Infertility
b. Methimazole b. Small testis
c. PTU c. Gynecomastia
d. Potassium perchlorate (KClO4) d. 47 XYY
e. Propranolol e. Phenotypically are males
64. A 35-years female presented with complaints of fatigue, cold 73. 22 years male presented with small testes and frequent erectile
intolerance and weight changes. On examination: dry skin, problems. He has moderate gynecomastia. The best investigation
thin brittle nails and bradycardia was observed. The most for this male would be
common cause of this presentation is: a. Semen analysis
a. Hashimoto’s thyroiditis b. Ultrasound of testes
b. Grave’s disease c. Karyotyping
c. Drug induced d. MRI of pituitary
d. Postpartum thyroiditis e. Testicular biopsy
e. Toxic multinodular goiter
21
DIABETES 8. In majority patients with type-2 diabetes are overweight and
1. What is the mechanism of action of dapagliflozin; drug used in obese. Which of the following would be most appropriate in the
the treatment of type 2 diabetes mellitus? (18LM Annual Exam) diabetes management with potential of causing weight loss?
a. DPP4 inhibitor a. Metformin & sitagliptin
b. SGLT2 inhibitor b. Metformin & rosiglitazone
c. GLP1 inhibitor c. Metformin & acarbose
d. Phenylalanine derivative d. Gliclazide & glargine
e. Alpha glucosidase inhibitor e. Liraglutide & dapagliflozin
2. Which of the following diabetes drugs act by decreasing the 9. A 55-years old patient of type-2 diabetes mellitus presented
amount of glucose produced by the liver? (18LM Annual Exam) with retinopathy of grade-2. Which of the following findings are
a. Sulfonylurea consistent for grade-2?
b. Meglitinides a. Vitreous detachment
c. Biguanides b. Macular edema
d. Alpha-glucosidase inhibitors c. Retinal hemorrhage
e. SGLT-II inhibitors d. Dot blot hemorrhage
e. Proliferative retinopathy
3. A 55-year-old woman is seen in clinic, she has a ten-year history
of type 2 diabetes treated with glipalamide. Her blood pressure is 10.A 55-years old diabetic female presents with repeated vaginitis
148/93 with new onset Proteinuria, A renal biopsy shows the due to which of the following anti-diabetic?
presence of Kimmelstiel-Wilson nodules. What is your a. Sitagliptin
appropriate management step in this patient? b. Empagliflozin
a. Calcium Channel Blockers c. Acarbose
b. Increase oral hypoglycemic dosage d. Lispro insulin
c. Start cholesterol lowering therapy e. Metformin
d. Start renal dialysis
e. Start ACE inhibitors 11.Which is NOT a feature of hyperosmolar hyperglycemic non-
ketotic states?
4. The rationale of palpating pulses in diabetic foot is: a. Lack of ketones in the urine
a. Assess ischemic foot b. Very high glucose, often greater than 400 mg/dl
b. Pulses are better indicator than ultrasound Doppler c. pH>7.3
c. Do daily and educate patient also d. Serum osmolality often greater than 350 mmol/L
d. Assess vascular patency e. Low bicarbonate
e. May require for antibiotics treatment
12.Which of the following anti-diabetic does not cause hypoglycemia?
5. Which is NOT true regarding the management of DKA? a. Sitagliptin
a. Cerebral edema responds to mannitol b. Repaglinide
b. Even if the K+ is >5.5 potassium replacement should be c. Glargine
commenced d. Pioglitazone
c. Fluid replacement should be 3-4 liters over the first four e. Glibenclamide
hours in a patient who is not shocked
d. The administration of IV fluid immediately on arrival is A 25-years old male presented with early morning hypoglycemia
the lifesaving event and repeated unconsciousness. Which of the following test would be
e. The insulin bolus should be about 0.1 units/kg with best for his diagnosis?
subsequent infusion at 0.1 units/kg/hr a. Serum cortisol level
b. Serum insulin level
6. A 72-years male known case of DM2 presented with urinary c. Thyroid profile
proteins 2+, which of the following drug is better for this d. C-peptide level
patient? e. HbA1C
a. Hydralazine
b. Losartan 13.A major disadvantage of the use of sulfonylureas in patients
c. Isosorbide dinitrate with diabetes is:
d. Atenolol a. Weight loss
e. Thiazide diuretic b. Vitamin B12 malabsorption
c. Risk of hypoglycemia
7. A 20-year-old woman presents with a 3-day history of diffuse acute d. Treatment cost
abdominal pain. On examination, skin turgor is reduced and a e. Risk of urinary tract infections
fruity odour can be smelt. The most likely diagnosis is:
a. Maple syrup urine disease 14.A 45-years old man is diagnosed as a case of type-2 diabetes. What
b. Pancreatitis is the minimum time period after which the HbA1c should be
c. Diabetic ketoacidosis repeated?
d. Acute porphyria a. 01 month
e. Liver failure b. 03 months
c. 06 months
d. 02 weeks
e. 04 months
22
15.Which of the following results establishes a diagnosis of 22.A 35-years old diabetic presented with unconsciousness. His
diabetes mellitus? blood sugar was 40mg. You have started I/V glucose but after 02
a. Symptomatic patient with fasting glucose 6.8 mmol/L on hours blood glucose level was 50 mg. What will be your next step
two occasion T of
b. Glycosuria +++ treatment?
c. Asymptomatic patient with fasting glucose 7.9 mmol/L on a. Start oral glucose as well as I/V replacement
one occasion b. Continue same treatment
d. Asymptomatic patient with random glucose 22.0 mmol/L on c. Give glucagon injection
one occasion. d. Start mannitol to treat cerebral edema
e. Symptomatic patient with random glucose 250 mg/dl on e. Start I/V adrenaline
one occasion.
23.Diagnosis of diabetes mellitus is confirmed by:
16.A 25-years old Asian woman, obese, who is 26 weeks pregnant a. Fasting plasma glucose > 100 mg/dl
has an OGTT. A recent ultrasound shows that the fetus is large for b. Fasting plasma glucose 116 mg/dl
age. OGTT reveals 280 mg/dl blood glucose. What is the most c. Fasting plasma glucose > 126 mg/dl on two different occasions
appropriate management: d. Fasting plasma glucose 136 mg/dl
a. Give advice about a diabetic diet + repeat OGTT in 04 weeks e. Fasting plasma glucose 146 mg/dl
b. Start gliclazide
c. Give advice about a diabetic diet 24.Clinical features of diabetes mellitus include all, EXCEPT:
d. Start insulin a. Up going planters
e. Start metformin b. 3rd nerve palsy with pupil sparing
c. Postural hypotension
17.Which one of the following statements regarding maturity d. Euphoria
onset diabetes of the young (MODY) is true: e. Weight gain
a. Body mass index is typically >30
b. There is usually a strong family history 25.Regarding diabetic ketoacidosis:
c. Doesn't respond to glimepiride a. Urinary acetones are negative
d. Autosomal recessive inheritance b. Blood sugar 250 gm/dl, pH 7.4, hyponatremia
e. Frequent episodes of diabetes ketoacidosis are typical c. It is metabolic alkalosis with pH 7.5
d. Initially presents with hyperkalemia with increased bicarbonate
18.For the treatment of obese diabetic patient, the most e. Serum uptake is a good indicator of prognosis
appropriate drug is:
a. Metformin 26.A 45-years old non-diabetic lady presented with
b. Vildagliptin unconsciousness, high grade fever. On examination no focal
c. Insulin neurological findings or neck rigidity elicited. Her laboratory
d. Glibenclamide report shows:
e. Glimepiride Serum Na 145 mEq/L
K+ 4 mEq/L
19.The sulphonylurea act by: Bicarbonate 24 mEq/L,
a. Decreasing the secretion of insulin Urea 84 mg/dl,
b. Increasing sensitivity of target tissues Creatinine 1.9 mg/dl and
c. Blocking the insulin receptors Random blood sugar 900 mg/dl.
d. Increasing the release of insulin from the pancreatic ẞ-cells The most likely diagnosis is:
e. Increasing glucagon secretions a. Meningitis
b. Diabetic ketoacidosis
20.Regarding sexual problems in diabetes: c. Cerebral malaria
a. Erectile dysfunction is seen between age group 35-59 years d. Encephalitis
in 75% e. Non-ketotic hyper osmolar coma
b. Hypertension do not contribute to sexual dysfunction in diabetes
c. Erectile dysfunction is due to endothelial dysfunction 27.A 57-years old man complains of increased thirst and urination.
d. A therapeutic trial of sideroff citrate is required in His examination is normal except for obesity. Which of the
most important diabetics following is the most appropriate initial diagnostic test?
e. Serum testosterone as-t prelastia are considered a. Fasting blood glucose
in hypergonadism b. Urine osmolality
c. Hemoglobin A1C level
21.A 30-years old diabetic presented with fluctuating sugar level. d. Glucose tolerance test
He claimed that he always taking his medicines properly. What is e. Random glucose
the tool to assess his sugar level?
a. C Peptide level 28.A 55-years old obese woman complains of vulvular Pruritis,
b. HbA1c recent weight loss in spite of a large appetite and waking up
c. Serum insulin level frequently in the night to urinate. Which of the following is the
d. Serum triglycerides most likely diagnosis?
e. Urine for protein a. Diabetes mellitus (DM)
b. Diabetes insipidus (DI)
c. Vaginitis & cystitis
d. Myxedema
23
e. Pheochromocytoma
24
29.Best investigation for diagnosis of diabetic nephropathy is: d. Pure ischemic ulcer represents 30% of foot ulcer
a. Microalbuminuria e. Neuropathy causes ulcer in 50% of cases.
b. U/S abdomen
c. Serum urea 37.A 50-year-old lady belongs to diabetic family brought her fasting
d. Serum creatinine blood report 124mg/dl and random blood sugar 190g/dl. What
e. CT scan abdomen is the next investigation for her problem?
a. Oral glucose tolerance test
30.A 39-years old male patient newly diagnosed with type-II b. Observation
diabetes mellitus. He is not on any medication. Which of the c. Oral hypoglycemic agent
following are true regarding clinical presentation, EXCEPT? d. Insulin therapy
a. Hyperosmolar non-ketotic coma e. Dietary modification
b. Carbuncle
c. Polyuria & polydipsia 38.Which is a Glucagon Like peptide 1 to treat diabetes?
d. Neuropathic ulcer a. Liraglutide
e. Hypoglycemia b. Sitagliptin
c. Glimepiride
31.A 46-years old man with type-II diabetes mellitus since 10 years d. Pioglitazone
presented with stable edema. Following lab reports confirm diabetic e. Ripaglitinide
nephropathy:
a. Urine analysis shows high leukocyte count 39.A 55 years diabetic male presents for uncontrolled blood
b. Albumin excretion rate of more than 300 mg/24 hours sugar. Which of the following is your target to maintain HbA1c
c. Urine analysis shows high RBC count level?
d. Urine analysis shows hyaline cast a. 6%
e. Urine analysis shows Bence Jones protein b. 8%
c. 6.5%
32.The clinical presentation in diabetic ketoacidosis (DKA) is: d. 9%
a. Hyperglycemia which leads to glycosuria & dehydration e. 10%
b. It is possible for patient to maintain hydration by stimulation
of thirst 40.A patient with type I diabetes mellitus is having convulsions in the
c. Increased lipolysis leads to ketone production including Emergency Department. Finger prick capillary glucose is 30
acetone mg/dl. At present there is no intravenous access. Which one of the
d. Decreased in pH causes rapid acidotic breathing called following is the most appropriate immediate management?
Chyne- stroke breathing a. Intramuscular adrenaline
e. Abdominal pain & vomiting results from acidosis b. Intramuscular glucagon
c. Oral glucose
33.Which one of the following is best in diabetic ketoacidosis? d. Subcutaneous 10% dextrose solution
a. In young adults the average total body water is about 2 liters e. Subcutaneous insulin
b. Insulin should be given by subcutaneous route
c. Intravenous bicarbonate is usually given if the arterial pH 41.A 15-years old boy who is diabetic presents with abdominal pain,
is below 5.1 vomiting and shortness of breath. There is history of fever and
d. Neutrophil leukocytosis can be present with infection sore throat two days back. The most likely cause of his symptoms
e. Hypothermia at presentation is a poor prognostic sign is:
a. Diabetic ketoacidosis
34.A 50-years old diabetic was taking glibenclamide for 05 b. Pancreatitis
years. Recently he was brought to casualty department for c. Hypoglycemia
sudden unconsciousness. What urgent test will you do? d. Non-ketotic hyperosmolar coma
a. Urine DR e. Renal failure
b. ECG
c. Chest X-ray 42.Which of the following tests is used for screening of
d. Blood sugar level gestational diabetes?
e. Fundoscopy a. RBS
b. HBA1c
35.A 25-years old male presented with increased blood pressure. c. Fructose amine
On examination BP 160/100 mmHg. fasting blood sugar was 140 d. 75g glucose tolerance test
mg and HbA1C is 7%. What other reports will you suggest? e. Urine for glucose
a. C. peptide level
b. Oral glucose tolerance test 43.Oral drugs used in management of DM with renal failure include:
c. Serum insulin level a. Metformin
d. Serum cortisol level b. Glibenclamide
e. Thyroid hormones analysis c. Pioglitazone
d. Acarbose
36. In Diabetic foot ulcer. e. SGLT2 inhibitors
a. Wegner classification is most commonly used
b. Infection is primary cause of foot ulcer
c. Autonomic Neuropathy causes insensitivity,
altered proprioception, small muscle wasting.
25
44.For diagnosis of DM, following 100g glucose load, 2-hours NUTRITION
RBS should be:
1. 25-hydro-oxylation of vitamin D occurs at the following site:
a. 110 mg/dL
a. Bones
b. 125 mg/dL
b. Liver
c. 150 mg/dL
c. Intestine
d. 180 mg/dL
d. Kidney
e. >200 mg/dL
e. Parathyroid gland
13.A young lady has presented with tingling sensation of the hands
and feet for last 02 to 03 months. On examination she is anemic
with no splenomegaly or lymphadenopathy. There is history of
worm infestation in the past. Her blood CP shows hemoglobin 7.9-
gram
percent with large RBCs. What test will you advise to this patient:
a. Vitamin D level
b. Serum calcium level
c. Serum iron level
d. Serum B12 level
e. Thyroid profile
27
HAEMATOLOGICAL DISEASES 8. A 20-year-old healthy boy was found to have a HIV positive
status during screening of blood, when he wanted to donate
1. A 32-year-old male presented with weakness and pallor. blood to be:
Laboratory report shows Hb 6.6gm/dl, reticulocyte count 8.0%. a. Start treatment of HIV
Which of the following feature confirm intravascular hemolysis? b. Repeat blood screening for HIV
(18LM Annual Exam) c. Repeat testing by ELISA
a. Free plasma hemoglobin d. Reassurance
b. Hemoglobinemia e. Check CD4 counts
c. Decrease haptoglobin
d. Methemoglobinemia 9. A 22-year-old man with sickle cell anemia presents with
e. Splenomegaly vomiting, abdominal pain and rash. He has been infected with a
parvovirus
2. Myeloproliferative disorder includes: (18LM Annual Exam) Blood results show Hb 4.5 g/dl and Reticulocyte count 3%. What is
a. Chronic myeloid leukemia the likely diagnosis?
b. Iron deficiency anemia a. Transformation to myelodysplasia
c. Primary polycythemia b. Thrombotic crisis
d. Paroxysmal nocturnal hemoglobinuria c. Sequestration crisis
e. Relative polycythemia d. Hemolytic crisis
e. Aplastic crisis
3. A young lady presented with fatigue and pallor within 03 days.
O/E anemia positive, spleen was just palpable. Her blood sent for 10.A 15-years old boy presented with fever and petechiae all over
cross matching and for complete picture. Lab technician find the body for last 02 days. There is no splenomegaly. Hb 12 g/dl,
difficulty during cross matching and her peripheral smear showed WBC 6000 and platelet counts 80000. What is the most likely
spherocytes. She is suffering from: diagnosis:
a. Sideroblastic anemia a. Thrombotic thrombocytopenic purpura
b. Hemolytic anemia b. Acute leukemia
c. Cold agglutinin disease c. Idiopathic thrombocytopenic purpura
d. Sickle cell anemia d. Aplastic anemia
e. Drug reaction e. Hemolytic uremic syndrome
4. A 65-years aged farmer presented with low grade fever and 11.A 42-years old female is noted to have hemoglobin 17.8 g/dl.
bleeding gums. On examination he is anemic with purpuric spots all Which one of the following is least likely cause:
over body and moderate spleen. Blood picture reveals a. Polycythemia rubra vera
pancytopenia and a dry tap on bone marrow aspiration. Most b. Chronic obstructive pulmonary disease
appropriate c. Hemolytic anemia
diagnosis will be: d. Hypernephroma
a. Acute lymphoblastic leukemia e. Dehydration
b. Aplastic anemia
c. Hypersplenism 12.The following peripheral smear finding is NOT matched to
d. Myelofibrosis correct clinical condition:
e. Megaloblastic anemia a. Hyper-segmented neutrophils Myelofibrosis
b. Basophilic RBC stippling myelodysplasia
5. One of the following statements is TRUE for activated c. Rouleaux formation paraproteinemia
partial thromboplastin time (aPTT): d. Acanthocytes chronic liver disease
a. It assesses extrinsic pathway of coagulation e. Howell-jolly bodies sickle cell disease
b. It helps in monitoring heparin therapy
c. It is low in antiphospholipid syndrome 13.A young male presents with anemia, bleeding and infection.
d. It is normal in hemophilia Blood count shows pancytopenia. The most valuable diagnostic
e. It is decreased in Von Willebrand disease investigation in this case is:
a. Coagulation screen
6. A 14-year-old boy with hemophilia A presented with spontaneous b. Plasma LDH
hemarthrosis in left knee joint. What treatment will you offer to this c. Complete blood count
patient? d. Bone marrow examination
a. Desmopressin e. CT scan chest & abdomen
b. Fresh frozen plasma
c. Cryoprecipitate 14.A 62-years old man presents with lethargy. A full blood count
d. Recombinant factor 8 concentrate is taken. Hb 10.2 g/dl, platelets 330x10°/L, WBC 15.2x10/L.
e. Mega unit of platelets Peripheral film shows tear drop cells and Poikilocytes. What is the
most likely diagnosis?
7. Which of the following is the most effective treatment a. Chronic lymphocytic leukemia
for Thrombotic thrombocytopenic purpura? b. Myelofibrosis
a. IV immunoglobulin c. Myelodysplasia
b. Whole blood transfusion d. Chronic myeloid leukemia
c. Platelet transfusion e. Post-splenectomy
d. Hemodialysis
e. Plasmapheresis
28
15.Which of the following blood film findings would be suggestive 23.Before going for tooth extraction in a patient. Who has been
of hyposplenism? treated for hepatitis B in the recent past. You will advise following
a. Basophilic stippling & cabot rings tests, EXCEPT:
b. Howell jolly bodies & siderocytes a. Blood CP
c. Rouleux formation b. PT
d. Schistocytes & eosinophilia c. Bleeding time
e. Toxic granulation & Dohle bodies d. Clotting time
e. Urea
16.A 25-years old patient presents with fatigue and pallor. She is found
to have iron deficiency anemia. Iron therapy is started. The 24.The best single test to confirm iron deficiency anemia is:
patient's response is monitored by: a. Total iron binding capacity (TIBC)
a. Serum ferritin b. Plasma transferrin
b. Serum transferrin c. Plasma ferritin
c. Serum iron d. Plasma iron
d. TIBC e. Transferrin saturation less than 16%
e. Reticulocyte count
25.Which of the following investigation confirms the iron deficiency?
17.A 22-years old pale looking female has the following lab results: Hb a. Microcytic hypochromic anemia
8.7 gm/dl, RBC microcytic hypochromic, target cells present, serum b. Low serum iron
iron. What is the diagnosis? c. Serum TIBC increased
a. Aplastic anemia d. Low serum ferritin level
b. Chronic alcoholism e. Erythroid hyperplasia in bone marrow
c. Folate deficiency anemia
d. Thalassemia minor 26.The best investigation to diagnose iron deficiency anemia is:
e. Anemia secondary to hemolysis a. Normal MCV
b. Low TIBC level
18.Which one of the following is the consistent with aplastic anemia? c. Normal serum iron level
a. Reduced marrow iron stores d. Low serum ferritin level
b. Moderate splenomegaly e. Reticulocytosis
c. Markedly reduced absolute neutrophil count
d. Hepatomegaly 27.In hypersplenism:
e. Lymphadenopathy a. Bi-cytopenia is usual
b. Extravascular hemolysis
19.A 21-years old man recently had an abdominal U/S for episodic c. Hemolysis is complement-mediated
right upper quadrant pain which demonstrated gallstones. d. Normal plasma volume
Blood film shows Hb 9.8 gm/dl, MCV 91fl, platelets 177x109/L, e. Pancytopenia is seen
WBC
5.3x109/L. Which test is most likely diagnostic? 28.Massive splenomegaly is NOT a feature of one of the
a. EMA binding test following diseases:
b. Direct Coombs test a. Chronic myeloid leukemia
c. PAS staining of erythrocytes b. Kalazar
d. Glucose-6-phosphate dehydrogenase levels c. Myelofibrosis
e. Ham's test d. Portal hypertension
e. Thalassemia
20.Cause of blood loss leading to iron deficiency anemia:
a. Worldwide most prevalent cause is hookworm infestation 29.In which of the following diseases massive splenomegaly is NOT
b. Breastfeeding in normal female a characteristic feature?
c. Use of steroids a. Kala-azar
d. Hematemesis b. Polycythemia rubra vera
e. Irritable bowel disease c. Thalassemia
d. Infectious mononucleosis
21.Deep vein thrombosis can be treated by: e. Chronic myeloid leukemia
a. Bed rest
b. Warfarin 30.Macrocytosis is NOT seen in:
c. Stockings a. Alcohol excess
d. Low molecular weight heparin b. Hypothyroidism
e. Aspirin c. Liver disease
d. Reticulocytosis
22.Prolong thrombin time in a patient having e. Thalassemia
bleeding problem indicates:
a. Functional platelet disorder
b. Decreased platelet count
c. Increased platelet count
d. Intrinsic pathway defect
e. Extrinsic pathway defect
29
31.Physiological cause of iron deficiency anemia: 40.A 10-years old boy was given Co-trimoxazole for fever. After
a. Gastric ulcer few hours he developed red color urine. The likely abnormality
b. Infancy is:
c. Jaundice a. G6PD deficiency
d. Menopause b. Malaria
e. Normal pregnancy c. Sickle cell disease
d. Hemophilia
32.Which of the following is NOT a symptom of anemia? e. DIC
a. Breathlessness
b. Tiredness 41.Malabsorption leading to iron deficiency anemia in:
c. Light headedness a. Hyper-chlorhydria
d. Chest pain b. One to H2 blockers use
e. Abdominal pain c. Gastritis
d. Celiac disease
33.Clinical features of anemia may include: e. Hypo-chlorhydria in young
a. Systolic flow murmur
b. Bradycardia 42.In aplastic anemia bad prognosis is associated with two of
c. Jaundice the following three features:
d. Clubbing a. Neutrophil count of <8x106/L, platelet count <10x
e. None of above 106L, reticulocyte count <40x109/L
b. Neutrophil count <0.5x109/L, platelet count <20x 109/L,
34.Which of the following is the feature of iron deficiency anemia? reticulocyte count <40x109/L
a. Hb 07 gm% c. Neutrophil count <5x109/L, increased cellularity of
b. Raised liver enzymes bone marrow, platelet count <20x109/L
c. Low plasma ferritin d. WBC <2x109/L, platelet count <90x109/L, reticulocyte count
d. Normocytic hypochromic anemia <30x109/L
e. Normal serum transferrin saturation e. WBC <2.5x109/L, platelet count 15x109/L. reticulocyte count
<40x109/L
35.Which one of the following characteristic feature of iron
deficiency anemia? 43.A 30-years old man had presented with history of lethargy and
a. Clubbing easy fatigability, he also complains of rashes on the body.
b. High serum ferritin Investigation reveals pancytopenia on blood CP, low reticulocyte
c. Low serum iron binding capacity count and bone marrow shows hypocellularity. Based on these
d. 20% reticulocyte count findings the most appropriate diagnosis is:
e. Microcytic hypochromic picture a. Aplastic anemia
b. Drug induced hemolytic anemia
36.Best treatment of iron deficiency anemia is: c. Hemolytic anemia
a. Blood transfusion d. Pure red cell aplasia
b. Ferrous sulphate e. Sickle cell anemia
c. Iron containing foods
d. Folie acid supplement 44.A 35-years old woman attends her regular medical clinic. The
e. Multivitamins physician notes that she is anemic and further investigation
reveals the following results:
37.Deficiency of vitamin B12 lead to: Hb = 6.5 g/dl
a. Microcytic anemia MCV = 76 fl
b. Macrocytic anemia MCH = 28 pg.
c. Hemolytic anemia MCHC = 33 g/dl
d. Sickle cell anemia WBC = 8.5x109/L with a normal differential,
e. Thalassemia Platelets = 505x109/L
Serum iron = 12 mmol/L
38.286. Which of the following is not a cause of vitamin B12 deficiency: TIBC = 40 mmol/L
a. Vegetarians Ferritin = 250 micro g/L
b. Pernicious anemia
c. Fish tape worm infection What is the most likely cause of her anemia?
d. Crohn's disease a. Sideroblastic anemia
e. Peptic ulcer T b. Iron deficiency anemia
c. Thalassemia trait
39.Which of the following is confirmatory test for macrocytic anemia? d. Anemia of chronic disease
a. Low Hb e. Lead poisoning
b. Low platelet count
c. Increased serum iron level
d. Reticulocyte count increase
e. Megaloblasts in bone marrow
21
0
45.An 18-years old male is recently diagnosed with beta- 53.A 20-year-old vegetarian male presented with fatigue and
thalassemia minor. His Hb is 10.5 g/dl and he feels well. Which lethargy. CBC reveals HB 6g/dl, WBC 3000 and platelet count
one of the following findings is characteristic of this condition? 100,000.
a. An increased amount of fetal Hb or hemoglobin A2 Peripheral neutrophils are hyper-segmented. What is the most
b. Increased osmotic fragility of the red cells likely diagnosis?
c. Absent bone marrow iron a. Acute leukemia
d. Increased macroglobulins in serum b. Aleukemic leukemia
e. Small amount of HbS c. Aplastic anemia
d. Iron deficiency anemia
46.A 63-years old man has stools positive for occult blood. e. Megaloblastic Anemia
Colonoscopy reveals mass in the sigmoid. colon. Which of the
following type of anemia is common in the patients? 54.All of the following cause Warm antibody hemolytic
a. Anemia of chronic disease anemia, EXCEPT?
b. Iron deficiency anemia a. Methyl dopa
c. Megaloblastic anemia b. SLE
d. Sickle cell anemia c. CLL
e. Pernicious anemia d. Paroxysmal nocturnal hemoglobinuria
e. Lymphoma
47.A 30-years old female presented with excessive menstrual bleeding.
She was using estrogen pills since 03 years. O/E patient was anemic 55.Which of the following is associated with Coomb’s
and purpuric rash over her skin. On investigation fragmented red negative hemolytic anemia?
cells seen on peripheral smear, LDH 300, in serum creatinine 60 a. Methyl dopa
mg/dl, Coombs test negative (-ve). What is the diagnosis of choice? b. SLE
a. Autoimmune thrombocytopenia c. CLL
b. Hemolytic uremic syndrome d. Paroxysmal nocturnal hemoglobinuria
c. Thrombotic thrombocytopenic purpura e. Lymphoma
d. Aplastic anemia
e. Evan's syndrome 56.Anemia of chronic disease is characterized by:
a. Anemia is usually severe.
48.Aplastic anemia is diagnosed by: b. Transferrin saturation value is always normal
a. Leukopenia c. Low serum iron with low to low-normal total iron-binding
b. Platelet count less than 20,000/dl capacity
c. Reticulocyte count less than 2% d. CRP level is diagnostic
d. Pancytopenia e. Decreased bone marrow iron stores
e. Hypocellular bone marrow
57.Anemia with low reticulocyte count and dry bone marrow
49.Regarding DIC select the best investigation: is suggestive of:
a. Thrombocytopenia a. Thalassemia
b. Raised PT & APTT b. Aplastic anemia
c. Decreased fibrinogen level c. Leukemia
d. Fragmented red cell d. Hemolytic anemia
e. Raised FDP e. Iron deficiency anemia
50.A 35-years old multigravida presented with weakness and paleness. 58.A 15-year-old male with certain anemia presents with left sided
On examination leukonychia, koilonychias, stomatitis, smooth abdominal mass and a hyperpigmentation. CBC shows: Hb 4.6
tongue with gross anemia were positive. What investigation would g/dl. Reticulocytes 3%, and severe poikilocytosis. What is the
you advise: likely diagnosis?
a. Upper GIT endoscopy alone a. Sickle cell crises
b. Bone marrow aspirate b. Thalassemia major
c. Colonoscopy c. Thalassemia minor
d. Hemoglobin electrophoresis d. Iron deficiency anemia
e. Complete blood picture, serum ferritin, TIBC e. Megaloblastic anemia
51.A 50-years old patient presented with aplastic anemia. Which of 59.A 35-years old female presented with palpitations, easy fatigability.
the following is appropriate treatment? Her work up shows iron deficiency anemia. She was started on oral
a. Steroids (corticosteroids) iron supplements. How much duration of iron therapy is required
b. Hematopoietic growth factor (filgestrime) to replenish iron stores:
c. Bone marrow transplantation & HLA identical sibling a. 02 Months
d. Repeated blood transfusions b. 03 Months
e. Immunosuppressive therapy c. 06 Months
d. 01 Year
52.Extravascular hemolysis leads to: e. 02 Years
a. Increased plasma haptoglobin
b. Lymph adenopathy
c. Splenomegaly
30
60.In a patient of thalassemia peripheral blood film for red 67.A 33-year-old male patient with a history of recurrent nose
cell morphology shows: bleeds, iron-deficiency anemia and dyspnea is found to have a
a. Hypochromic microcytic cells pulmonary AV-malformation on pulmonary angiography. What is
b. Normochromic normocytic cells the
c. Macrocytosis underlying diagnosis?
d. Sickle cells a. Hemophilia A
e. Spherocytes b. Hereditary hemorrhagic telangiectasia
c. Wegener’s granulomatosis
61.In polycythemia rubra vera: d. Down’s syndrome
a. A transient blurring of vision is impossible
b. The tendency for thrombosis only affects the veins 68.A 24-years old woman presented with recent epistaxis. She is also
c. The Budd-Chiari syndrome develops having heavy periods. On examination bruises were present over
d. Gastrointestinal bleeding can occur both legs. Her laboratory report shows WBC 8000/cmm, Hb%
e. The development of gout is a rare complication 11.5 gm/dl, platelet 20000x10/1, antinuclear antibody negative.
The most likely diagnosis is:
62.A 30 years male presented with pallor. CBC reveals a Hb 7gm/dL, a. Chronic liver disease
MCV 116Fl, peripheral smear shows macrocytosis with hyper- b. Aplastic anemia
segmented neutrophils. What is the cause of anemia in this c. Idiopathic thrombocytopenic purpura
patient? d. Thrombotic thrombocytopenic purpura
a. Folic acid deficiency e. Acute leukemia
b. Hypothyroidism
c. Chronic liver disease 69.A 60 years old male presented in ER with stroke, he doesn’t
d. Alcohol intoxication have any comorbid. Her lab workup reveals Hb 12 g/dL, WBCs
e. Aplastic anemia 11000, platelet count of 1200,000. What is most likely
diagnosis?
63.A 15-year-old boy presented in OPD with anemia and jaundice. a. Polycythemia rubra vera
His CBC shows Hb 7gm/dL with Heinz bodies and bite cells on b. Essential thrombocytosis
peripheral smear. LFTs show indirect bilirubin levels of 4mg/dl. c. Chronic myeloid leukemia
He was diagnosed with Plasmodium vivax malaria 10 days back. d. Secondary thrombocytosis
What is the most likely diagnosis? e. Myelofibrosis
a. Malaria
b. Warm agglutinin hemolytic anemia
MEDICAL ONCOLOGY INCLUDING HAEMOTOLOGICAL
c. G-6-PD deficiency
MALIGNANCY
d. Sickle cell trait
e. Cold agglutinin hemolytic anemia
1. Which of the following is NOT a cutaneous marker of
internal malignancy? (18LM Annual Exam)
64.Which of the following is most specific for polycythemia rubra vera?
a. Bulous pemphigoid
a. CBC with peripheral smear
b. Red cell mass b. Acanthosis nigricans
c. Serum erythropoietin level c. Dermatomyositis
d. Janus kinase 2 mutation d. Erythema chronicum migrans
e. Hematocrit levels e. None of the above
65.Which of the following is a cause of extravascular hemolysis? 2. Which of the following condition is an example of
a. Hereditary spherocytosis paraneoplastic syndrome?
b. G-6-PD deficiency a. Irritable bowel syndrome
c. DIC b. Hypoglycemia
d. Mismatched blood transfusion c. Variceal hemorrhage
d. Dilated pupil
e. Hemolytic uremic syndrome
e. Hypercalcaemia
66.A 14-years boy presents with a three-day history of throbbing pain
and increasing difficulty breathing at night. Physical examination; 3. A young patient presents with features of anemia, neutropenia
in addition to wheezing reveals petechiae over abdomen. Chest X- and thrombocytopenia. A large number of blasts are present on
ray reveals normal and CBC shows platelets 50,000/cmm. The most bone marrow biopsy. Which investigation would help differentiate
likely diagnosis is: between acute myeloid leukemia (AML) and acute lymphoblastic
a. Atypical pneumonia leukemia (ALL)?
b. ITP a. Sudan black
c. DIC b. Auramine stain
d. TTP c. Myeloperoxidase stain
d. Tartrate-resistant acid phosphatase stain
e. Acute leukemia
e. Leukocyte alkaline phosphatase
32
5. An 18-years old male is diagnosed as a case of leukemia. Which 12.In chronic myeloid leukemia common sign is:
one of the following is characteristic feature of acute leukemia? a. Hepatomegaly
a. Slow onset of fever & anemia b. Lymphadenopathy initially
b. Mouth ulceration & gingival hypertrophy c. Pallor & jaundice
c. Myalgia, arthralgia & skin rashes are absent d. Massive splenomegaly
d. Microcytic anemia & leukopenia e. Skin nodules
e. Hypocellular bone marrow cytology
13.Chronic myeloid leukemia is diagnosed by:
6. An 18-years young man has presented with high grade fever, skin a. Increased leukocyte count
rash and body ache for last six weeks. On examination he is b. Presence of Philadelphia chromosome
looking pale with mild splenomegaly and cervical c. Increased platelet count
lymphadenopathy. His d. Increased level by uric acid
blood CP shows Hb 6.9gm%, WBC 177000/cmm3 and platelets e. Elevated LDH level
45000/cmm3 and 42% blast cells. What is likely diagnosis?
a. Acute myeloid leukemia 14.A 25-years young man has presented with generalize
b. Lymphoma lymphadenopathy and fever for last one month. On examination
c. Chronic myeloid leukemia patient is vitally stable with temperature 100°F. Lymph nodes are
d. Aplastic anemia mobile, discrete and non-tender. Lymph node biopsy shows Reed
e. Acute lymphoblastic leukemia Sternberg cell positive. The likely diagnosis is:
a. CML
7. A 60-years old male patient presents with pain and mass in b. Non-Hodgkin lymphoma
left lumbar region. On examination he has a plethoric face. His c. Hodgkin lymphoma
CBC shows Hb 20%. What is the most likely diagnosis? d. Acute leukemia
a. Renal cell carcinoma e. Tuberculosis
b. Ca pancreas
c. Ca bladder 15.Which one of the following conditions, is associated with
d. Ca colon acute promyelocytic leukemia?
e. Ca prostate a. DVT
b. DIC
8. A 50-years old man presents with early satiety, splenomegaly c. Myocardial infarction
and bleeding. Cytogenetics show t(9,22). What is the most likely d. Pulmonary infarction
diagnosis? e. Hyper-viscosity syndrome
a. CML
b. CLL 16.A 30-years old man is investigated for enlarged, painless cervical
c. ALL lymph nodes. A diagnosis of Hodgkin's lymphoma is made.
d. AML Which one of the following classifications is useful?
e. Lymphoma a. Child-Pugh
b. Revised American
9. A 60-years old female was presented with bleeding mouth. On c. Rai-staging
examination anemia was presented and liver spleen was d. International working
palpable. On investigation TLC count was 60 thousand/cmm, e. Ann arbor
platelet count was 30 thousand. Peripheral smear showed
nucleated RBC and some promyelocyte and myeloblasts. Which is 17.Which of following sign is mostly observed in chronic
likely diagnosis: myeloid leukemia?
a. Myelodysplasia a. Hepatomegaly
b. Chronic myeloid leukemia b. Lymph adenopathy
c. Multiple myeloma c. Deep jaundice
d. Myelofibrosis d. Retinal haemorrhage due to leukocytosis
e. Acute myeloid leukemia e. Massive Splenomegaly
10.Clinical manifestations of bone marrow failure are: 18.While looking around a blood smear, you notice a blast with an
a. Reticulocytosis Auer rod in it. This patient has:
b. Bone fractures a. Bacterial infection
c. Repeated infections b. No disease, unless 20% of the nucleated cells have Auer rods
d. Mild anemia c. A myelodysplastic syndrome
e. Muscle wasting d. Acute myeloid leukemia
e. Acute lymphoblastic leukemia
11.In chronic myeloid leukemia following signs are seen:
a. Pallor & jaundice
b. Splenomegaly often moderate
c. Lymphadenopathy initially T
d. Retinal hemorrhage due to leucostasis
e. Hepatomegaly
33
19.A 50-year-old patient who is recently diagnosed as squamous
cell carcinoma of lung presented with early morning headache
and puffiness of face for last 1 week. On examination he has
severe
puffiness of face with conjunctival congestion. What should be the
most immediate step in the management?
a. IV diuretics
b. Radiotherapy
c. Palliative Chemotherapy
d. Stenting
e. Urgent pneumonectomy
34
RHEUMATOLOGICAL DISORDERS e. Dermatomycosis
1. Which of the following feature is NOT commonly found in 8. A 26-years old female presented with acute pain in right hip joint
CREST syndrome? (18LM Annual Exam) with high-grade fever and diarrhea for seven days. Her CBC shows
A. Eczema leukocytosis; uric acid level, renal function & urine DR were
B. Oesophageal dysfunction normal. The most likely test for the diagnosis would be:
C. Raynaud's phenomenon a. MRI of hip joint
D. Sclerodactyly b. CRP & ESR
E. Telangiectasia c. X-ray of hip joint
d. Stool culture & CBC
CREST— Calcinosis cutis, anti-Centromere antibody, Raynaud’s e. HLA-B27
phenomenon, Esophageal dysfunction, Sclerodactyly, Telangiectasia
9. A 55-years old female patient who is a known case of systemic
2. Which of the following is UNLIKELY feature of Henoch- sclerosis, suffered from fingertip ulceration and necrosis and pain.
Schoenlein purpura syndrome? (18LM Annual Exam) The most likely diagnosis is:
A. IgA nephropathy a. Primary Raynaud’s phenomenon
B. Palpable purpura b. Secondary Raynaud’s phenomenon
C. Arthritis c. Buerger's disease
D. Abdominal pain d. Intermittent claudication
E. Thrombocytopenia e. Acute limb ischemia
3. Which are extra-articular features of Ankylosing spondylitis? (18LM 10.The ocular symptoms in Rheumatoid arthritis are:
Annual Exam) a. Red eye
A. Nasal septum deviation b. Episcleritis
B. Posterior uveitis c. Dryness of eye
C. Lower lobe fibrosis d. Rheumatoid nodule
D. Upper lobe fibrosis e. Scleritis
E. Aortic stenosis
11.The hematological complications of Rheumatoid arthritis are:
4. One of anti-tubercular drugs is responsible for hyperuricemia a. Eosinophilia
that leads to gouty arthritis (18LM Annual Exam) b. Lymphopenia
A. Ethambutol c. Microcytic anemia
B. Isoniazid d. Thrombocytosis and splenomegaly
C. Pyrazinamide e. Anemia
D. Rifampicin
E. Streptomycin 12.A 22-years old female presented with multiple sites pain over the
joints for 2 years after divorced. On examination there were
5. A 70-years old woman with a history of vertebral crush many tender points. Her routine investigation including CBC,
fractures presents to the osteoporosis outpatient clinic. Which multiple bone X-rays, CRP, uric acid were normal. The most likely
of the following investigations is most useful to assess the extent diagnosis is:
of her osteoporosis? a. Autoimmune disease
a. MRI scan b. Reactive arthritis
b. Vitamin D levels c. Polymyalgia rheumatic
c. Full blood count, bone & liver biochemistry blood tests d. Fibromyalgia
d. DEXA scan e. Rheumatoid arthritis
e. Spinal X-rays
13.A knee joint effusion D/R of 70 years old male shows
6. A 51-years old female referred by her GP over concern for calcium pyrophosphate crystals. Your diagnosis is:
osteoporosis. She had a hysterectomy and oophorectomy because a. Gout
of uterine fibroids one year ago after which she developed hot b. Osteoarthritis
flushes. Her mother had a history of fracture of femoral neck. She is c. Rheumatoid arthritis
worried she can develop this. She is smoker and drink 05 units of d. Pseudogout
alcohol e. Septic arthritis
weekly. On examination she has a BMI of 18, BP 122/88 mmHg and
other examination is normal. Her hip BMD is -2.7. Which of the 14.A 40-years female known case of rheumatoid arthritis presented
following would you recommend for her? with increasing joint stiffness, fever and ill health for 02 years, she
a. Combined estrogen & progesterone therapy is taking Prednisolone, diclofenac, allopurinol, methotrexate and
b. Bisphosphonates tramadol for 2 years. The rheumatologist wants to start tofacitinib.
c. Calcitonin The most likely risk of starting a biological agent would be:
d. Vitamin D a. Reactivation of tuberculosis
e. SERM b. Nephropathy
c. Gastritis
7. Anti-CCP antibody is positive in: d. Severe headache
a. SLE e. Osteomalacia
b. Rheumatoid arthritis
c. Psoriasis
d. Systemic sclerosis
35
15.A 30-years old woman complains of intolerance to cold water and 22.A middle age lady newly diagnosed as case of RA started
dysphagia. On examination she has beaked nose, fish mouth, facial DMARDs by physician. After 6 months of therapy, she developed
telangiectasia, sausage like digits and tapered fingers. Which one diarrhea,
is the best test to diagnose this disease? loss of hairs, and dramatic weight loss. LFT was disturbed. Which
a. RA factor drug is culprit?
b. Blood CP a. Sulfasalazine
c. X-ray chest b. Leflunomide
d. Anticentromere antinuclear antibody (Dx. CREST syndrome) c. Rituximab
e. Barium swallow d. Methotrexate
e. Chloroquine
16.Disease which presents as inflammatory monoarthritis is:
a. Reactive arthritis 23.A 50-years old male presented with difficulty in hearing and
b. Psoriasis nasal septum perforation. His B.P was 130/100 mmHg. His urine
c. Ankylosing spondylitis D/R
d. Septic arthritis showed red cells casts, protein 2+, Chest X-ray had multiple
e. Post viral arthritis opacities. The most likely cause is:
a. Tuberculosis
17.Main clinical criteria for the diagnosis of Rheumatoid arthritis: b. Amyloidosis
a. Morning stiffness > 03 hours c. Good pasture syndrome
b. Arthralgia of three or less joints d. Kawasaki disease
c. Symmetrical arthritis of more than 6 weeks e. Wagener's granulomatous
d. Asymmetrical arthritis
e. Urethritis 24.A 35-years old female presented with joint pain and swelling for 01
year that has increased in early morning hours. On examination
18.Methotrexate is used for: she has nodules on extensor aspect of forearm. What is the likely
a. The only disease modifying agent to treat R.A diagnosis?
b. It is safe in pregnancy a. Osteoarthritis
c. It is the drug of choice by many physicians b. Septic arthritis
d. Oral vitamin B6 is given with it to counter its side effects c. Chondro-calcinosis
e. It is given monthly d. Rheumatoid arthritis
e. SLE
19.Following drug is included as disease modifying anti-
rheumatic drugs (DMARDs): 25.Which of the following is noticed in sarcoidosis?
a. Corticosteroids a. Elevated uric acid level
b. NSAIDs b. Hyperphosphatemia
c. Tetracycline c. Hypo-gammaglobunemia
d. Leflunomide d. Increased level of serum calcium
e. Infliximab e. Leukopenia
20.A 19-years college student having H/O polyarthralgia, rash over 26.A 50-years old woman who has received a recent diagnosis of
her cheeks. On examination reveal generalized edema with mild rheumatoid arthritis, presents to her GP with ongoing pain
right and
sided pleural effusion. Investigations show pancytopenia and stiffness in her hands and feet. Which joints are usually spared at
proteinuria of 2+. The best test to assess active lupus nephritis will onset of rheumatoid arthritis?
be: a. Metatarsophalangeal joints
a. ESR estimation b. Distal interphalangeal joints
b. GFR estimation c. Proximal interphalangeal joints
c. Anti-DNA antibody levels d. Metacarpophalangeal joints
d. Serum-complement levels e. Wrists
e. Renal biopsy
27.Which of the following medication causes rhabdomyolysis?
21.A 50-years old woman presented with a two-day history of a. Phenytoin
increasing difficulty with swallowing and regurgitation of food. Over b. Simvastatin
the preceding two three months she had also experienced difficulty c. Carbamazepine
climbing stairs, raising her arms above the head and arising from a d. Sumatriptan
sitting or lying position with some associated lower back pain. On e. Fusidic acid
examination she was found to have symmetrical proximal muscle
weakness with normal tone, sensation and deep tendon reflexes. 28.Bouchard's and Heberden's nodules are feature of one of
Cranial nerve examination revealed weakness of the bulbar muscles. the following:
What is the most likely cause of her muscle weakness? a. RA (rheumatoid arthritis)
a. Polymyositis b. MCD (mixed connective tissue disorder)
b. Motor neuron disease c. AS (ankylosing spondylitis)
c. Myasthenia gravis d. OA (osteoarthritis)
d. Hypothyroidism e. SLE (systemic lupus erythematous)
e. Poly-neuralgia
36
29.A 55-years old type-II diabetic male presented with pain in right 37.The most appropriate diagnostic test for rheumatoid arthritis is:
shoulder for 03 months. On examination patient could not raise a. ANA
the arm above 40. The most likely diagnosis is: b. Raised C-reactive protein
a. Pseudogout c. Anti-CCP
b. Drug induced arthritis d. Rheumatoid factor
c. Gout e. Raised ESR
d. Osteoarthritis
e. Frozen shoulder 38.A young male presents with fever, dysuria, conjunctivitis
and arthritis involving lower limbs. There is history of
30.A 73-years old male presented with an acute attack of gout in attack of dysentery 02 weeks before. The most likely
his left knee. What is the most likely underlying cause? diagnosis is:
a. Endogenous overproduction of uric acid a. Behcet's disease
b. Lactic acidosis b. Felty's syndrome
c. Excessive dietary protein intake c. Septic arthritis
d. Starvation d. Reiter's syndrome
e. Decreased renal excretion of uric acid e. Gonococcal arthritis
31.Which one of the following tests is most specific for SLE? 39.Which one of the following statements is TRUE to
a. Anti-RO ankylosing spondylitis?
b. Anti-dsDNA a. No sec: dissemination
c. Anti-cardiolipin antibodies b. HLA-B27 is positive in 100%
d. ANA c. Achilles tendonitis is common
e. Anti-LA d. Schober's test is diagnostic
e. Osteolytic lesions are common
32.A 40-years old male presents with painful, swollen big toe.
Serum uric acid is raised. Synovial fluid examination is likely to 40.A 60-years old female patient presents with severe headache and
reveal? scalp tenderness. She also has fever, fatigue and weight loss.
a. Normal synovial fluid Examination reveals thickened and tender temporal arteries. In
b. Needle like crystals, negatively birefringent this patient what would you advise:
c. Positive birefringent crystals a. NSAIDS
d. Many neutrophils & gram-positive bacilli b. Morphine
e. Many neutrophils & gram-negative bacilli c. Corticosteroids
d. Azathioprine
33.Myasthenia gravis is NOT associated to which of the following? e. Tryptanol
a. Rheumatoid arthritis
b. Systemic lupus erythematosus 41.In which of the following is an increased risk of atlanto-
c. Thyroid disease axial dislocation with spinal cord injury?
d. Hyperparathyroidism a. Subdural hematoma with raised intracranial pressure
e. Pernicious anemia b. Ankylosing spondylitis
c. Rheumatoid arthritis
34.A known hypertensive presented with severe pain in great toe. On d. Psoriatic arthropathy
examination big toe is red, edematous, tender and hot. He is using e. Gout
thiazide diuretics for his BP control. Which investigation would
you advise for diagnosis? 42.Patient having rheumatoid arthritis can develop
a. Serum creatinine levels following pulmonary complications, EXCEPT:
b. X-ray of big toe a. Ca. lung
c. Antinuclear antibody b. Pleural effusion
d. Blood CP c. Pulmonary fibrosis
e. Serum uric acid level d. Pulmonary tuberculosis
e. Pulmonary hypertension
35.A middle-aged lady is on methotrexate for rheumatoid arthritis.
She is likely to develop: 43.A 16-years old girl presents with 03 months history of
a. Folate deficiency polyarthralgia and marked early morning stiffness. Her symptoms
b. Vitamin B12 deficiency responded well to diclofenac sodium but she is becoming increasing
c. Vitamin D deficiency concerned about her symptoms which appears to be progressing.
d. Iron deficiency She is otherwise well apart from history of acne which is well
e. Calcium deficiency controlled on minocycline. Investigations; ESR: 50mm/hour, RA
factor negative, ANA positive, Anti-ds-DNA negative and Anti-
36.A 62-years old woman presents with the symptoms of dry eyes and histone antibodies positive. What is the most likely diagnosis:
dry mouth. She has arthritis as well. Which of the following a. Spondyloarthropathy
arthritis she is likely to be suffering from? b. SLE
a. Rheumatoid arthritis c. RA
b. Gonococcal arthritis d. Fibromyalgia
c. Pseudo-arthritis e. Drug induced SLE
d. Osteoarthritis
e. Gouty arthritis
37
44.A 35-years old female complaining of increasing widespread joint d. Mononeuritis
pain with morning stiffness lasting for more than 01 hour for last e. Pleuritis
04 months. On examination she has swelling of wrist at PIP and
MCP with deformities. RA factor is positive, anti-CCP is positive. 52.A 25-year-old female presented with stroke. Her attendant gives
Which of the following statement is CORRECT? history of repeated leg swelling in past. She has livedo
a. NSAIDs with steroids reticularis. What test you will advise?
b. Prednisone 60mg per day should be started a. Anti-cardiolipin test (dx. Anti-phospholipid syndrome)
c. NSAIDs along with steroids with DMARDs b. RA factor
d. Physiotherapy alone c. Anti SSA
e. NSAIDs alone d. Ski to
e. Anti CCP
45.Which of the following is included in diagnostic criteria
of Rheumatoid arthritis? 53.A 50-year-old diabetic presented with non-healing ulcer on right
a. Asymmetrical & migratory arthritis foot. Following feature is suggestive of peripheral arterial
b. Arthritis of sacroiliac joint disease?
c. Arthritis of foot joints a. Absence of vibration sense
d. Arthritis of 03 or more joints b. Loss of position sense
e. Morning stiffness < 30 minutes c. Shiny & cold skin
d. Loss of calf muscles
46.The seronegative spondyloarthropathies include: e. Loss of ankle reflexes
a. Ankylosing spondylitis
b. Rheumatoid arthritis 54.In Ankylosing spondylitis, the clinical manifestations are:
c. Gonococcal arthritis a. Chronic backache in old age, worse in morning
d. Systemic lupus erythematosus b. Chronic low backache in younger age, worse in morning
e. Infections arthritis c. Conjunctivitis in 25% cases
d. The pain & stiffness does not improve with activity
47.A 45-years old female presented with fever, skin ulcer e. The pain always radiates into buttocks
and symmetrical arthropathy since last 04 years. She is
taking 55.The extra articular neurological manifestation of
ibuprofen, allopurinol for 01 year. On examination she is pyrexial, rheumatoid arthritis leading to cord compression and
mild splenomegaly and CBC shows neutropenia. Her best diagnosis sudden death are:
is: a. Sudden neck flexion causing cord compression in
a. Drug induced arthritis atlantoaxial subluxation
b. Felty's syndrome b. Compression neuropathies
c. Reactive arthritis c. Carpel tunnel syndrome
d. Pseudogout d. Mo-neuritis multiplex
e. Osteoarthritis e. Cervical cord compression
48.A 10-years old boy presented with fever, maculopapular rash and 56.A 50-years old man with joint pain, muscle pain, esophageal
joint pain. On examination temperature was 39° C and dysmotility with +ve ANA. What should be your best
generalized lymphadenopathy. His lab reports show; negative RA- diagnosis?
factor and ANA. What is the most likely diagnosis? a. Scleroderma
a. Septic arthritis b. SLE
b. Still's disease/Systemic juvenile idiopathic arthritis c. Mixed connective tissue disease
c. Rheumatoid arthritis d. Polymyositis
d. Gout e. RA
e. Osteomalacia
57.An elderly woman presents with pain and stiffness of the
49.The biologic DMARDs used in rheumatoid arthritis are: shoulder joint and hips. She can't lift her arms above her head.
a. Gold Lab study
b. Azathioprine shows anemia and increased ESR. What is most likely diagnosis?
c. Leflunomide a. Polymyalgia rheumatica
d. Adalimumab b. RA
e. Methotrexate c. Osteoarthritis
d. Ankylosing spondylitis
50.A joint effusion shows rhomboidal weakly birefringent.
Your diagnosis will be: 58.A 55-years old man has sudden excruciating first MTP joint pain
a. Gout after a night drink of red wine. What is the most likely
b. Rheumatoid arthritis diagnosis?
c. Septic arthritis a. Gout
d. Pseudo-gout b. Septic arthritis
e. Osteoarthritis c. Osteoarthritis
d. RA
51.The vascular symptoms in rheumatoid arthritis are:
a. Vasculitis 59.Regarding Temporal arteritis:
b. Glomerulonephritis a. Occur in the patient at the age of 30 years
c. Scleritis
38
b. Main complaint is throbbing unilateral headache
c. Involvement of extra ocular muscle
d. ESR is not raised
e. Corticosteroid are contraindicated
39
60.A 60-years old female with the complaint of joint pain and 68.A 30-years old housewife presents with dysphagia and difficulty
stiffness that worsens over the end of the day and are relieved by in opening the mouth. She has history of Raynaud's phenomenon.
rest. What would be the diagnosis? On examination her skin is inelastic and thick. What is the most
a. Osteoarthritis likely diagnosis?
b. RA a. Sjogren syndrome
c. Gout b. Systemic sclerosis
d. Polymyalgia rheumatica c. Dermatomyositis
e. Pseudogout d. Polymyositis
e. SLE
61.Rheumatoid arthritis usually involves:
a. Distal inter-pharyngeal joints 69.A 20-years old female presented with hematuria. O/E she has
b. Proximal inter-pharyngeal joints alopecia, loin tenderness, soft systolic murmur and oral ulcers.
c. Knee joint Her blood CP shows leukopenia and thrombocytopenia. How will
d. Shoulder joint you
e. Sacroiliac joint confirm the diagnosis?
a. Echocardiography
62.Ankylosing spondylitis is: b. Antinuclear antibody with anti-ds antibody
a. Common after 50 years c. Blood culture
b. Common in females d. Renal biopsy
c. Revisable disease e. HIV antibody
d. Characterized by low backache
e. Best treated with steroids 70.A patient presented with back ache and legs weakness since last 2
months. On Examination there was a bony deformity at dorsal
63.Rheumatoid arthritis is: spine after this illness. What is your diagnosis?
a. An infectious disease a. Potts spine
b. An autoimmune disorder b. Spinal stenosis
c. More common after 60 years of age c. Frederic Ataxia
d. Reversible disease d. Tabes dorsalis
e. All of above e. Transverse myelitis
64.A 20-years old female is diagnosed as a case of SLE. Which one 71.A 63-years very sick male due to chronic diarrhea presented due to
of the following is a characteristic clinical feature? bony pain since last 3 months. On examination there was
a. Raynaud's phenomenon & photosensitivity generalize bony tenderness and X-ray of femur showed ‘looser
b. Alopecia & livedo reticularis are absent zone’. What would be the best test for his diagnosis?
c. Erosive asymmetrical large joint arthritis a. DEXA scan
d. Fibrosing alveolitis & glomerulonephritis b. MRI of spine
e. Pericarditis & endocarditis c. Tumor markers for metastasis
d. Serum calcium, phosphorus and vitamin D
65.A 72-years old female is diagnosed as a case of osteoarthritis. e. PSA
Which one of the following is best regarding this condition?
a. Ankle joint is commonly involved 72.A 65-years old man presents with a sudden, persisting
b. Distal interphalangeal joint involvement is uncommon monocular vision loss. There is a history of continual ipsilateral
c. Thoracic spine involvement is common headache for the past 12-months. What laboratory test is most
d. X-ray shows narrowing of joint spaces relevant to the likely diagnosis?
e. ESR is characteristically elevated a. Urinalysis
b. Complete blood count
66.A 20-years old male presents with sudden severe pain in the left c. Erythrocyte sedimentation rate
knee. Synovial fluid aspirate shows negatively birefringent d. Chest X-ray
needle shaped crystals under polarized light. What is the most e. Cerebral CT-scan
likely diagnosis?
a. Gout
73.A 55-years old female presents with low-grade fever,
b. Pseudogout generalized body ache and thigh pain since last two years. Her
c. Septic arthritis all routine- investigations were normal except CRP which was
d. Osteoarthritis
55 mg/dL
e. Tuberculosis of joint (normal; 6 mg/dL). The most likely diagnosis is:
a. Fibromyalgia
67.Which one of the following statements regarding arthritis is TRUE?
b. Polymyositis
a. Septic arthritis is usually polyarticular
c. Polymyalgia rheumatica
b. Rheumatoid arthritis typically involves the
d. Osteoarthritis
terminal interphalangeal joints
e. Rheumatoid arthritis
c. Gouty arthritis predominantly affects middle aged females
d. Rheumatoid nodules carry a good prognosis
74.In Osteoarthritis clinical manifestation would be:
e. Morning stiffness is a feature of inflammatory arthritis
a. Non deforming arthritis
b. Bouchard’s nodules
c. Proximal myopathy
d. Sacro-iliac joint tenderness
31
0
e. Well preserved joint movements
31
1
75.A 14-years old boy is brought to a medical OPD with 3. A patient presents with non-suppurative, painful, palpable
palpable lesions over his buttocks. He is also passing dark nodular lesions at shins. He is diagnosed case of inflammatory
urine with bowel
occasional abdominal pain. He is most probably suffering from? disease. The best diagnosis of these lesion is:
a. Poly arteritis nodosa a. Purpura
b. Henoch Schoenlein purpura b. Dug induced eruption
c. IgA nephropathy c. Erythema multiform
d. Nephrotic syndrome d. Erythema nodosum
e. Sickle cell disease e. Eczema
76.A 55-year-old banker finds increasing difficulty to climbing his 4. The drug of choice for head lice is:
office stairs daily. He is also not able to swallow food with a. Topical steroids
pain. b. Antihistamines
There is a peculiar rash around his eyes. Most helpful investigation c. Pyrethrin & malathion
in solving the problem is: (dx. Dermatomyositis) d. Retinoic acid
a. Anti Jo-1 antibody e. Antibiotics
b. Anti-DNA antibody
c. ANA 5. The most characteristic feature of erythema multiforme is:
d. Biopsy of quadriceps muscle a. Pustules
e. Electromyography of quadriceps muscle b. Wheals
c. Target lesions
77.Which of the following infectious organism is the most d. Bullous lesion
common cause of acute infectious arthritis in adults? e. Severe itch
a. Group B streptococci
b. Neisseria gonorrhea 6. A 14-years old girl presents with nocturnal itching for 02 weeks.
c. Propionibacterium acnes Her 06 months old brother has itchy papulo-pustular eruptions
d. Staph. aureus on palms and soles. The most likely diagnosis is:
e. Echinococcus granulosus a. Urticarial
b. Pemphigus
78.Which of the following is the common initial manifestation c. Scabies
of ankylosing spondylitis? d. Tineasis
a. Acute iritis e. Contact dermatitis
b. Hemoptysis 7. The most common type of vitiligo is:
c. Hip arthritis a. Segmental vitiligo
d. Inflammatory back pain b. Focal vitiligo
e. Aortic regurgitation c. Generalized vitiligo
d. Mucosal vitiligo
79.Regarding drug-induced SLE: Select best one e. Abdominal vitiligo
a. Renal and CNS involvement is common
b. Not resolved after stopping drug 8. All of the following are causes of photosensitivity. EXCEPT:
c. Double stranded DNA is specific for diagnosis a. SLE
d. Antibodies to specific drug are present b. Porphyria
e. Most common drugs causing SLE are: Procainamide, c. Drugs
Isoniazid, Hydralazine d. Xeroderma pigmentosa
e. Scabies
DERMATOLOGY
1. A 20-year-old boy presented with skin lesion over the dorsal 9. A 23-years old receptionist presents with dome- shaped papules on
aspects of his right foot and was diagnosed to have pyoderma arms, wrists, dorsa of hands and lower back since 03 months. The
gangrenosum. Which of the following should be looked for? (18LM papules were circular to ovoid in outline, small in size, lilac-
Annual Exam) colored and associated with severe itch. The most likely diagnosis
a. Enteric fever would be:
b. Sarcoidosis a. Chicken pox
c. Tuberculosis b. Darier's disease
d. Ulcerative colitis c. Lichen planus
e. Kawasaki disease d. Lichen planus scabies
e. Psoriasis
2. A 55-years old man has non-infective necrotizing ulcers on his
lower limbs. A diagnosis of pyoderma gangrenosum is made. 10.A 40-years old female presents with a six months history of
Which of the following condition is associated with this disorder? pruritic eruptions, vesicles and excoriations on the elbows, knees,
a. Inflammatory bowel disease buttocks and scalp. What is the most likely diagnosis?
b. Hemochromatosis a. Atopic dermatitis
c. Gout b. Dermatitis herpetiformis
d. non-Hodgkin’s lymphoma c. Henoch Schoenlein purpura
e. Autoimmune thrombocytopenia d. Psoriasis
e. Scabies
31
2
11.A young man from flood affected area is brought to the local 6. Regarding depression:
doctor for generalized Itching increased during night. He also a. An uncommon
reported b. It comprises sadness & helplessness
same problem in other family members. What is the treatment? c. It is not a medical problem
a. Anti-allergic d. Depression is not treatable
b. Anti-helminthic e. Depression is not manageable at home
c. Ivermectin
d. Psoralans 7. Regarding schizophrenia:
e. Phototherapy a. It comprises hallucinations & delusions
b. It comprises high sugar level
12.A 17-years old patient presents with severe pruritus that is worse c. It is not a medical problem
at night upon examination of the skin, areas of excoriated papules d. It is not a treatable illness
are observed in the interdigital area. Family members report e. Schizophrenia is a kind of depression
similar symptoms. The following is the most likely diagnosis:
a. Scabies 8. A 20-years old student was accompanied by his parents. His
b. Cutaneous larva migrans mother insisted that he along with a habit of repeated washing of
c. Contact dermatitis hands.
d. Dermatitis herpetiformis The anxiety is relieved after washing his hands. What is the most
e. Impetigo likely diagnosis:
a. Depression
13.Which are the best typical features of acne vulgaris, EXCEPT: b. Schizophrenia
a. Pilosebaceous glands & their ducts are not involved c. Bipolar personality disorder
d. Obsessive compulsive disorder
PSYCHIATRY e. Psychosis
1. Mania does NOT characterize by: (18LM Annual Exam) 9. A 20-years old male presents with a history of auditory
a. Elated moods hallucinations associated with delusion and mood
b. Over activities disturbance. What is the most likely diagnosis?
c. Joyfulness a. Obsessive compulsive disorder
d. No needs for sleep b. Conversion disorder
e. Psychomotor retardation c. Depression
d. Schizophrenia
2. A patient has a habit of repeated hand washing, washing hands e. Mania
for 50 times and avoids to everyone. He is a case of:
a. Obsessive compulsive disorder 10.Most effective drug for treatment of obsessive-compulsive
b. Persistently disorder disorder is:
c. Conversion disorder a. Imipramine
d. Mental disorder b. Clomipramine
e. Epileptic disorder c. Tranylcypromine
d. Amitriptyline
3. A 09-years old girl presented with history of sudden blindness e. Amoxapine
after watching her parents in a fight at home. She has normal
neurological findings. She is likely to be suffering from: 11.Hysteria is associated with the defense mechanism of:
a. Schizophrenia a. Acting out
b. Conversion disorder b. Denial
c. Mania c. Identification
d. Hypochondriasis d. Reaction formation
e. Depression e. Somatization
4. Which statement is suitable for suicide? 12.A farmer presented with emergency with hallucination and
a. Self-infliction shouting after taking cannabis. How will you classify this disorder?
b. Males complete more suicide a. Neurosis
c. Females attempt more suicide b. Psychosis
d. It is leading cause of death worldwide c. Depression
e. No medical management available d. Anxiety
e. Delirium
5. Regarding generalized anxiety disorders:
a. Denial 13.‘Perception without corresponding environmental stimuli’ is:
b. More common in complex urban societies a. A hallucination
c. More common in females b. An illusion
d. Precipitating trauma should be of extreme intensity which c. A delusion
is beyond normal human experience d. Derealization
e. Has high co-morbidity rates e. Depersonalization
40
DRUGS AND REACTIONS
41